Sie sind auf Seite 1von 146

RIFT VALLEY UNIVERSITY COLLEGE

CENTER FOR DISTANCE EDUCATION

RISK MANAGEMENT AND INSURANCE

(MGMT 333)

CREDIT HOUR: 3

Degree Program

December, 2012

Adama, Ethiopia
COURSE INTRODUCTION
Dear Learner! How are you? How are the courses you have taken in the previous semester?
And well come to fantastic course called Risk Management and Insurance. Risk affects
every aspect of an organization. The effects of risk are not confined within any predictable
boundaries; a single event can easily influence several areas of an organization at once,
producing consequences far beyond the immediate impact. The pervasiveness and complexity
of risk presents strong challenges to managers, one of the most important being the
coordination of risk management across areas within the organization. It deals with: the
nature and management of pure risks, insurance and reinsurance; risk concepts, classification
of risks, management of pure risks through various risk handling tools, industrial safety,
general principles of insurance and major classes of insurance, reinsurance and development
& regulation of the insurance industry in Ethiopia.

The module consists of about eight chapters;

Chapter One: Risk and Related Topics

Chapter Two: The Risk Management

Chapter Three: Insurance

Chapter Four: Legal Principle of Insurance Contract

Chapter Five: Life and Health Insurance

Chapter Six: Non-Life Insurance

Chapter Seven: Re-Insurance

Chapter Eight: The Insurance Business in Ethiopia


Course Objectives

After the completion of this course, the students will be able to:

 Know basic concepts of risk


 Explain the basic classification of risk
 Manage risk and identify the process of managing it as risk manager
 The course is intended to enable students to identify and measure business loss
exposures.
 Explain insurance; its meaning, principles, and types
 Discusses how to select among the major tools of risk management and quip
students with ways of measuring, if not eliminating, exposures to loss or risk and
the ability to analyze various class of insurance contracts
 Explain the nature and application of reinsurance
 Understand insurance Industry in Ethiopia
CHAPTER ONE
RISK AND RELATED TOPICS

Introduction

Dear learner, well come to Risk Management and Insurance course .The adverse effects of risk
have affected mankind since the beginning of time, individuals groups and societies have
developed various methods for managing risk. Since no one knows the future exactly, everyone
is a risk manager not by choice, but by absolute necessity. In this chapter, you will learn about
the fundamental concepts in risk and insurance and terms such as peril, hazard, probability,
uncertainty their relation with risk and their difference from risk. Finally we will discuss the
classifications of risk depending on different variables.
Objectives:
After studying this chapter, you should be able to:
 Define the term risk
 Distinguish risk from uncertainty.
 Distinguish risk from Chance of Loss.
 Discuss on the difference between risk, hazard and peril.
 Identify the classes of hazard
 Categorize risk in to different class. .
 Describe the categories in to which pure risk may be subdivided
1.1. Definitions of Risk

 Dear learner, would you define risk?

______________________________________________________________________________
The starting point for any reading material on risk and insurance must be the concept of risk
itself and our understanding of it. The word is certainly used frequently in everyday conversation
and seems to be well understood by those using it.

1
! There is no single definition of risk. Many writers have produced a number of definitions of

risk. These are usually accompanied by lengthy arguments to support the particular view they put
forward. Economists, behavioral scientists, risk theorists, statisticians, and actuaries each have
their own concept of risk. Some of these definitions are forwarded for your consideration as
follows.
1) According to William, Smith and Young, Risk is potential variation in outcomes. When risk is
present, outcomes cannot be forecasted with certainty.
2) Risk is the variation in outcomes that could accrue over a specified period in a given situation.
William’s and Heins
3) According to Vaughen, Risk is the condition in which there is a possibility of adverse deviant
from desired outcome that is expected or hoped for.
4) Williams and Heins did not focus only on the negative side as variation could be both positive
and negative. The emphasis is then on both negative and positive feelings of risk. But Vaughen
focuses on the negative felling of risk.
The value of having a single definition is questionable because it is likely to be limited in its
ability to capture the comprehensive flavor of risk. It is more valuable to dissect the idea of risk
and consider its component parts.

! Dear learner, despite the theorist’s disagreement on the single universally accepted definition,

there are common elements in all the definitions: Indeterminacy and Lo


1) Indeterminacy
The concept of indeterminate outcome is implicit in all the definition of risk, the outcome must
be in question. When risk is said to exist, there must always be at least two possible outcomes. If
we know certainly that a loss will occur, there is not risk. Example: Investment in capital asset,
plants and machinery. As a matter of fact these assets are subject to physical depreciation and
that their value will decline. Here, the outcome, the decline in value, is certain and therefore,
there is no risk.

2
2) Loss
As can be understood in the above concept, indeterminate outcome, there are at least two
possible outcomes. So as to declare the situation as risk, one of the possible outcomes must be
undesirable. This may be a loss in generally accepted sense or it may be a gain smaller than the
gain that was possible. Based on the above concepts risk can be defined as “a condition of the
real world in which there is an exposure to adversity”. More specifically, they define the term
risk as a condition in which there is possibility of an adverse deviation from a desired outcome
that is expected or hoped for. In addition, risk traditionally has been defined in terms of
uncertainty. Based on this concept risk is defined as “uncertainty concerning the occurrence of
loss”.
1.2. Risk Vs Uncertainty

 Dear learner, can you distinguish risk and uncertainty?

______________________________________________________________________________
In its working definition of risk, we said that it was uncertainty about the outcome in a given
situation. Uncertainty is at the very core of the concept of risk itself, but are we clear what we
mean by it when we use the word.

! The term “uncertainty” is often used in connection with the term risk; sometimes even it is

used interchangeably. Because of it seems appropriate to explain and differentiate these two
terms. The most widely held meaning of uncertainty refers to a state of mind characterized by
doubt, based on lack of knowledge about what will or will not happen in the future. Uncertainty,
then, is simply a psychological relation to the absence of knowledge about the future. In contrast
risk is a situation where there is a possibility of loss and no loss and even a possibility of profit
(although it is not always and in every risky situation). Examples of risky situations may include
becoming a business man (starting a business); playing any kind of gambling e.t.c can be a good
example of a risky situation.
We can therefore say that risk exists outside the individual, it may be recognized as existing but
this is not a pre-requisite. In this sense, it is objective and not dependent on any one individual.

3
In chapter two we will see that people very often do place their own subjective assessments on
the existence and level of risk in given situations.
1.3. Risk and Probability/Chance

 Dear learner, would you distinguish risk and probability?

______________________________________________________________________________
_________________________________________________________________________

! Chance of loss is closely related to the concept of risk. Chance of loss is defined as the

probability that an event will occur. Chance of loss should not be confused with objective risk.
Chance of loss is the probability that an event will occur. Objective risk is the relative variation
of actual loss from the expected loss. The chance of loss may be for two different groups, but
objective risk may be quite different.

For example, assume that a fire insurer has 10,000 homes insured in Adama and 10,000 homes in
Bishoftu and that the chance of loss in each city is 1 percent. Thus, on average, 100 homes
should burn annually in each city. However, if the annual variation in losses ranges from 75 to
125 in Adama, but only from 90 to 110 in Bishoftu, objective risk is greater in Adama even the
chance of loss in both cities is the same.

1.4. Risk, Peril and Hazard

 Dear learner, can you distinguish peril and hazard in risk?

______________________________________________________________________________
_____________________________________________________________________________

We often use the word risk is to mean both the event, which will give rise to some loss and the
factors which may influence the outcome of a loss. When we think about cause, we must be clear
that there are at least these two aspects to it. We can see this if we think of a house on a

4
riverbank and the risk of flood. The risk of flood does not really make sense, what we mean is
the risk of flood damage. Flood is the cause of the loss and the fact that one of the houses was
right on the bank of the river influences the outcome. Flood is the peril and the proximity of the
house to the river is the hazard.

! The peril is the prime cause of loss; it is what will give rise to the loss. Often it is beyond the

control of anyone who may be involved. In this way we can say that storm, earth quake, flood,
tornadoes, hail, fire, theft, motor accident and explosion are all perils.
Factors, which may influence the outcome, are referred to as hazards. Hazards refer to the
conditions that create or increase the chance of loss resulting from a given peril. These hazards
are not themselves the cause of the loss, but they can increase or decrease the effect should a
peril operate. In fact, hazards would facilitate the occurrence of perils. The consideration of
hazard is important when an insurance company is deciding whether or not it should insure some
risk and what premium to charge.

! There are three major types of hazards:

1. Physical hazards 3. Morale hazard


2. Moral hazard
1. Physical hazard is a physical condition that increases the likelihood of loss. It relates to the
physical characteristics of the item or the property exposed to the risk.

! Examples of physical hazard among others include the following;

 Defective wiring in a building that increase the chance of fire


 Defective locking on a door that increases the chance of theft.
 An icy road that increases the chance of automobile collusion
 Existence of dry forest that can increase the chance of fire to be happen
 loose security protection at a shop or factory, etc

5
2. Moral hazard is dishonesty or character defects in an individual that increases the frequency or
severity of loss. It is related with the human aspects which may influence the outcome. This
usually refers to the attitude of the insured person.

! Examples of moral hazard among others include;

 Submitting a fraudulent claim


 Inflating amount of claim
 Intentionally burning unsold merchandise that is insured.
A dishonest person in the hope of collecting from insurance company may intentionally cause
accident or exaggerate the amount of a loss in an attempt to collect more that the amount that
he/she is entitled. The losses that can result from such evil actions of individuals are considered as
moral hazard.
Those evil actions of insured’s emanates from the ground that “the insurer has plenty of money”
this view is incorrect because the insurer can pay claims only by collecting premium from other
insured’s. Because of this premium will be high for everyone.
3. Morale hazard refers to the carelessness or indifference to a loss because of the existence of
insurance. Some insured’s are careless or indifferent to a loss because they have insurance.

! Examples of morale hazard among others include:

 Leaving car keys in an unlocked car, which increases the chance of theft;
 Leaving a door unlocked that allows a burglar to enter, etc…
Morale hazard can also be reflected in attitude of persons other than insured’s. The tendency of
physicians to provide more expensive level of care when cost are covered by insurance;
similarly, the inclination of judges to makes larger awards when the loss is covered by insurance
is another example for moral hazard.

 Activity 1

1) Define risk

6
_______________________________________________________________________________
2) Discuss the differences between peril and hazard in risk
_______________________________________________________________________________
_______________________________________________________________________________
_______________________________________________________________________________
3) List and explain types of hazard by giving examples
_______________________________________________________________________________
_______________________________________________________________________________

1.5. Classification of Risk

 Dear leaner, would you guess how risk is classified?

______________________________________________________________________________
Now we see the classifications into which risk can be placed. This is different from scrutinizing
the actual idea of risk; we are now looking at the whole concept of risk and grouping together
similar classes of risk.

! Of the many classes, we will look at five classifications in this module.

1) Objective and Subjective Risks


Some authors make a careful distinction between objective risk and subjective risk. We shall
briefly discuss the distinction between the two below:

A. Objective risk: is defined as the relative variation of the actual loss from expected loss. For
example, assume that a fire insurer has 10,000 houses insured over a long period and, on average,
1 percent, or 100 houses burn each year. However, it would be rare for exactly 100 houses to burn
each year. In some years as few as 90 houses may burn, while in other years, as many as 110
houses may burn. Thus, there is a variation of 10 houses from the expected number of 100, or a
variation of 10 percent. This relative variation of actual loss from expected loss is known as
objective risk.

7
! Objective risk can be statistically measured by some measure of dispersion, such as the

standard deviation or the coefficient of variation. Since objective risk can be measured, it is an
extremely useful concept for an insurer or a corporate risk manager. As the number of exposures
increases, an insurer can predict its future loss experience more accurately because it can rely on
the law of large numbers. The law of large numbers states that as the number of exposure units
increases, the more closely will the actual loss experience approach the probable loss experience.
For example as the number of homes under observation increases, the greater is the degree of
accuracy in predicting the proportion of homes that will burn.

B. Subjective risk: is defined as uncertainty based on a person’s mental condition or state of


mind. For example, an individual is drinking heavily in a bar and attempts to derive home. The
driver may be uncertain whether he or she will arrive home safely without being arrested by the
police for drunk driving. This mental uncertainty is called subjective risk. Often subjective risk is
expressed in terms of the degree of belief.

The impact of subjective risk varies depending on the individual. Two persons in the same
situation may have a different perception of risk, and their conduct may be altered accordingly. If
an individual experiences great mental uncertainty concerning the occurrence of a loss, that
person’s conduct may be affected. High subjective risk often results in less conservative conduct,
while low subjective risk may result in less conservative conduct. A driver may have been
previously arrested for drunk driving and is aware that he or she has consumed too much alcohol.
The driver may then compensate for mental uncertainty by getting someone else to drive him or
her home or by taking a cab. Another driver in the same situation may perceive the risk of arrested
as slight. The second driver may drive in more careless and reckless manner; a low subjective risk
results in less conservative driving behavior.

2) Financial and Non-Financial Risks


We have already said that risk implies a situation where there is uncertainty about the outcome. A
financial risk is one where the outcome can be measured in monetary terms. This is easy to see in
the case of material damage to property, theft of property or lost business profit following a fire.

8
In cases of personal injury, it can also be possible to measure financial loss in terms of a court
award of damages, or as a result of negotiation between lawyers and insurers. In any of these
cases, the outcome of the risky situation can be measured financially.

There are other situations where this kind of measurement is not possible in terms of money. Take
the case of the choice of a new car, or the selection of an item from a restaurant menu. These
could be taken as risky situations, not because the outcome will cause financial loss, but because
the outcome could be uncomfortable or disliked in some other way. We could even go as far as to
say that the great social decisions of life are examples of non-financial risks: the selection of a
career, the choice of a marriage partner, having children, the death of bread winner of the family.
There may or may not be financial implications, but in the main the outcome is not measurable
financially but by other, more human, criteria.

! In the world of business we are primarily concerned with risks which have a financially

measurable outcome. Even if there are also non-financial risk in this module great emphasis is
given for financial risk.

3) Pure and Speculative Risks


This classification also concerns the result of outcome of the loss. It distinguishes between those
situations where there is only the possibility of loss and those where a gain may also result.
A. Pure risks involve two possible outcomes a loss or, at best, no loss. The outcome can only be
unfavorable to us, or leave us in the same position as we enjoyed before the event occurred.
Examples of pure risks:
 The risk of a motor accident
 fire at a factory
 theft of goods from a store
 Injuries at work are all pure risks with no element of gain.

! It is a loss or no loss that can result from such risks.

9
The major types of pure risks that are associated with great financial and economic insecurity
include personal risks, property risks, and liability risks.

i. Personal risk is chiefly concerned with death and the time of its occurrence. And apart from
death, there is incapacity through accident, injury, illness or old age – loss of earning power.
Dear student, there are four major personal risks:
 Risk of premature death
 Risk of insufficient income during retirement
 Risk of poor health
 Risk of unemployment

ii. Property risk refers to losses associated with ownership of property such as destruction of
property by fire, lightening, windstorm, flood and other forces of nature. Property risk leads to
direct loss and consequential loss. For example, when the New York twin towers were destroyed,
the direct loss is the building itself and the consequential loss is the benefit generated from it
including the rent income.

! Losses to property may be classified as either direct loss or indirect loss. Each of this group

is discussed below.

a) Direct loss: is a direct loss is defined as a financial loss that results from the physical damage,
destruction, or theft of the property. For example, assume that you own a restaurant, and the
building is insured by a property insurance policy. If the building is damaged by a fire, the
physical damage to the property is known as a direct loss. In other words, property suffers a
direct loss when the property itself is directly damaged or destroyed or disappears because of
contact with a physical or social peril.

b) Indirect or consequential loss: is a financial loss that results indirectly from the occurrence
of a direct physical damage, destruction, or theft. Thus, in addition to the physical damage loss,
the restaurant would lose profits for several months while it is being rebuilt. The loss of profits
would be a consequential loss. Other examples of consequential loss would be the loss of the use

10
of the building, the loss of rents, and the loss of a market. Extra expenses are another type of
indirect or consequential loss. For example, suppose you own a newspaper, bank, or dairy. If a
loss occurs, you must continue to operate regardless of cost; otherwise, you will lose customers
to your competitors. It may be necessary to set up a temporary operation at some alternative
location, and substantial extra expenses would then be incurred.
iii. Liability Risk
Liability risk is the possibility of loss arising from intentional or unintentional damage made to
other persons or to their property. One would be legally obliged to pay for the damages he/she
inflicted upon other persons or their property. A court of law may order you to pay substantial
damages to the person you have injured.

! Liability risks are of great importance for several reasons.

First, there is no maximum upper limit with respect to the amount of the loss. You can be sued
for any amount. In contrast, if you own a property, there is a maximum limit on the loss. For
example, if your automobile has an actual cash value of Br. 10,000, the maximum physical
damage loss is Br. 10,000. But if you are negligent and cause an accident that results in serious
bodily injury to the other driver, you can be sued for any amount – Br. 50,000, Br. 500,000, or
Br.1 million or more – by the person you have injured.

Second, although the experience is painful, you can afford to lose your present financial assets,
but you can never afford to lose your future income and assets. Assume that you are sued and are
required by the court to pay a substantial judgment to the person you have injured. If you do not
carry liability insurance or are underinsured, your future assets can be attached to satisfy the
judgment. If you declare bankruptcy to avoid payment of the judgment, your ability to obtain
credit will be severely impaired.

Finally, legal defense costs can be enormous. If you are sued and have no liability insurance, the
cost of hiring an attorney to defend you and represent you in a court of law can be staggering.

11
B. Speculative risk: is the alternative to pure risks is speculative risk, where there are two
possible outcomes – gain or loss. Speculative risk is defined as a situation in which either profit
or loss is possible. Investing money in shares is a good example. The investment may result in a
loss or possibly a break-even position, but the reason it was made was the prospect of gain.
People are more adverse to pure risks as compared to speculative risks. In speculative risk
situation, people may deliberately create the risk when they realize that the favorable outcome is,
indeed, so promising.

! Dear student, it is important to distinguish between pure and speculative risks for three

reasons.
First, private insurers generally insure only pure risks. With some exceptions, speculative risks
are not considered insurable and other techniques for coping with risk must be used. The reasons
why speculative risks are uninsurable are:
1) Exception is that some insurers will insure institutional portfolio investments and municipal
bond against loss.
2) The law of large numbers can be applied more easily to pure risks than to speculative risks.
The law of large numbers is important since it enables insurers to predict losses in advance. In
contrast, it is generally more difficult to apply the law of large numbers to speculative risks in
order to predict future loss experience.
3) Society may benefit from a speculative risk even though a loss occurs, but it is harmed if a
pure risk is present and a loss occurs. For example, a firm may develop a new technological
process for producing computers more cheaply and, as a result, may force a competitor into
bankruptcy, society benefits since the computers are produced more efficiently and at a lower
cost. However, society will not benefit when most pure risks occur, as for example, if a flood
occurs or an earthquake devastates an area.

The reason for stressing the difference between pure and speculative risks is to highlight the fact
that pure risks are normally insurable while speculative risks are not normally insurable. It is
difficult to be dogmatic about this, as practice is changing and the division between pure and
speculative is becoming more unclear as time passes. Take the case of the credit risk, which we

12
listed under the heading of speculative risks. The goods have been sold on credit in the hope that
a gain will result but a form of credit insurance is available which will meet some of the
consequences should the debtor default.

However, insurance is not normally available for those risks where the outcome can be a gain
and it is easy to see why this should be so. Speculative risks are entered into voluntarily, in the
hope that there will be gain. There would be very little incentive to work towards achieving that
gain if it was known that an insurance company would pay up, regardless of the effort expended
by the individual. Using the terminology of hazard, we could say that there would be a very high
risk of moral or morale hazard.
However, we should be clear that the pure risk consequences of speculative risks can be insured
against and that more and more people involved in risk and insurance are being asked to handle
speculative risks.
4) Static and Dynamic Risks
A. Dynamic risk originates from changes in the overall economy such as price level changes,
changes in consumer tastes, income distribution, technological changes, political changes and the
like. They are less predictable and hence beyond the control of risk managers.
B. Static risks, on the other hand, refer to those losses that can take place even though there
were no changes in the overall economy. They are losses arising from causes other than changes
in the economy. Unlike dynamic risks, they are predictable and could be controlled to some
extent by taking loss prevention measures. Many of the perils fall under this category. Examples
of statistics include the uncertainties due to random events such as fire, windstorm, or death.
5) Fundamental and Particular Risks
The final classification relates to both the cause and effect of risk/consequences of the losses.
A. Fundamental risks are those, which arise from causes outside the control of any one
individual or even a group of individuals. In addition, the effect of fundamental risks is felt by
large numbers of people. This classification would include earthquakes, floods, famine,
volcanoes and other natural ‘disasters’. However it would not be accurate to limit fundamental
risk to naturally occurring perils. Social change, political intervention and war are all capable of
being interpreted as fundamental risks. In contrast to this form of risk, which is impersonal in
origin and widespread in effect, we have particular risks.

13
B. Particular risks: in contrast to fundamental risks, particular risk is a risk that affects only
individuals not the entire community. Examples of particular risk include car theft, bank
robberies and fire to a dwelling e.t.c. from such a loss only individuals will suffer loss not the
entire economy or community. Since fundamental risks are caused by conditions more or less
beyond the control of the individuals who suffer the losses and since they are not the fault of
anyone in particular it is held that society rather that than the individuals has a responsibility
to deal with them. Therefore social assistance like social insurance, government insurance
programs, as well as governmental guaranties and subsidies are necessary to solve problems
that are created because of fundamental risk exposures. Example: the risk of unemployment is
generally uninsurable by private insurance but it can be insured or assisted by publicly by state
unemployment program.

 Activity 2

1) Discuss the different classifications of risk


______________________________________________________________________________
____________________________________________________________________________
2) Liability risks are of great importance for several reasons. Mention those reasons and
elaborate them
______________________________________________________________________________
______________________________________________________________________________
3) List examples of particular and fundamental risks
______________________________________________________________________________
_____________________________________________________________________________

 Check List

Dear learner, below are the most important points which are drawn from the chapter you have
been studying up to now. Put a tick () mark in the box after the point you feel you have

14
understood. But if you find a concept that you have not mastered yet, please go back and read the
chapter you passed through.

I can:

1) Define risk

2) Distinguish uncertainty from risk

3) Distinguish risk, peril and hazard

4) List down and discuss type of hazards

5) List and explain the classification of risks

6) Differentiate pure risk and speculative risks

7) Give examples for each type of risks

Chapter Summary
There is no single definition of risk. Many writers have produced a number of definitions of risk.
These are usually accompanied by lengthy arguments to support the particular view they put
forward. Economists, behavioral scientists, risk theorists, statisticians, and actuaries each have
their own concept of risk. Risk is potential variation in outcomes. When risk is present, outcomes
cannot be forecasted with certainty. Risk is the variation in outcomes that could accrue over a
specified period in a given situation. Objective risk is the relative variation of actual loss from
expected loss. Subjective risk is uncertainty based on an individual’s mental condition or state of
mind. Chance of loss is defined as the probability that an event will occur; it is not the same
thing as risk. Peril is defined as the prime cause of the loss. Hazard is any condition that creates
or increases the chance of loss resulting from a given peril. There are three major types of
hazards. Physical hazard is a physical condition present that increases the likelihood of loss.
Moral hazard is dishonesty or character defects in an individual that increases the likelihood of
loss. Morale hazard is carelessness or indifference to a loss because of the existence of insurance.
The basic categories of risk include the following: objective and subjective risk, pure and
speculative risk, fundamental and particular risk, static and dynamic risk, financial and non-

15
financial risk. A pure risk is a risk where there are only the possibilities of loss or no loss. A
speculative risk is a risk where either profit or loss is possible.
A fundamental risk is a risk that affects the entire economy or large number of persons or groups
within the economy, such as inflation, earth quake, flood, war, or recession. A particular risk is a
risk that affects only the individual and not the entire community or country. The following
kinds of pure risk can threaten an individual’s financial security: Personal risks, Property risks,
and Liability risks

 Self Test Questions

Part-I: Say “True” for the correct statement and “False” for the incorrect statement
1) There is one universal accepted definition of risk.
2) Indeterminacy and loss are the two common terms in all definition of risks.
3) Particular risk is a type of risk that affects the entire economy or large group of societies with
the economy.
4) In pure risk there are the possibilities of loss or gain.
5) Subjective risk is a type risk that is measured in figures and expressed statistically.

Part-II: Choose the best answer among the given alternatives


1) Which of the following is not true about risk?
A. It is a potential variation in outcome.
B. Being an inherent element in business it cannot be managed.
C. It has something to do with future uncertainty.
D. All of the above
E. None of the above
2) All of the following are situations lead to personal risk, except?
A. Risk of premature death
B. Risk of poor health
C. Risk of unemployment
D. Risk of insufficient income during retirement
E. Property risk

16
3) Carelessness or indifference to a loss because of the existence of insurance is called_____
A. Peril C. Morale hazard
B. Moral hazard D. None
4) Which of the following peril is/are not included under fundamental risk?
A. Inflation D. Robbery
B. Earth quake E. Famine
C. War

5) Based on changes in economy risk is classified as:


A. Fundamental and particular risks
B. Static and dynamic risks
C. Pure and speculative risks
D. Objective and subjective risks

Part-II: Short Answer Questions


1. In your definition, state the relationship between risk and uncertainty?

2. Define peril and hazard in risk and give examples for each?

3. Distinguish the difference between pure and speculative risk?

4. What is the difference between direct and indirect loss?

17
CHAPTER TWO
THE RISK MANAGEMENT PROCESS

Introduction

Dear learner, the concept, which develops, is one of risk as an all-pervasive force in the world;
which has a negative feature in life. It brings unfortunate or unlooked outcomes. The various
classifications that we have used all tend to support the view that risk is to be managed at any
costs. In the first chapter of this course, we have identified several types of pure risks that affect
individuals and businesses. After sources of risks are identified and measured, a decision can be
made as to how the risk should be handled or managed. The process used to systematically
manage pure risk exposures is known as risk management. In this chapter we will see risk
management; objectives of risk management, steps to be followed in management of risk.

Objectives
After studying this chapter, you should be able to:
 Define risk management
 Explain objectives of risk management.
 Explain risk management process/steps
 Describe the basic components of risk management tools.
2.1. Risk Management Defined

 Dear learner, can you guess what mean by risk management?

____________________________________________________________________________

! The following definitions of risk management have been presented for your study.

18
Definition 1
Risk Management refers to the identification, measurement and treatment of exposure to
potential accidental losses.
Definition 2
Risk Management is defined as a systematic process for identification and evaluation of pure
loss exposures faced by an organization or individual, and for the selection and
implementation of most appropriate techniques for treating such exposures.

Definition 3

Risk Management is the executive function of dealing with specified risks facing the
business enterprise. In general, the risk manager deals with pure, not speculative risk.

! Form the above definitions we can understand that risk management is

 A systematic approach to deal with risk.


 It only deals with pure risk not speculative risks.
 All pure risks are considered including those which are not insurable.
Dear student, a risk manager has certain specific duties to be performed by him/her. These
include:
 To recognize exposures to loss; the risk manager must, first of all, be aware of the
possibility of each type of loss. This is a fundamental duty that must precede all other
functions.
 To estimate the frequency and size of loss; to estimate the probability of loss from
various sources.
 To decide the best and most economical method of handling the risk of loss, whether it be
by assumption, avoidance, self-insurance, reduction of hazards, transfer, commercial
insurance, or some combination of these methods.
 To administer the programs of risk management, including the tracks of constant
revaluation of the programs, record keeping and the like.

19
2.2. Objectives of Risk Management

 Dear student, would you imagine objectives of risk management?

______________________________________________________________________________

! Risk management objectives serves as a prime source of guidance for those charged with

responsibility for the program (risk managers). It also serves as a means of evaluating
performance.
Risk management has several important objectives that can be classified in to two categories
1. Pre-loss objectives
2. Post-loss objectives

OBJECTIVES

PRELOSS OBJECTIVES POSTLOSS OBJECTIVES

To prepare for the potential Survival of the firm


losses in the most economical
way
To continue operating
Reduction of anxiety

Stability of earnings
To meet any externally imposed
obligation
Continued growth

Social responsibility

20
1) Pre-loss objectives
There are various risk management objectives prior to the occurrence of loss. The most
important pre-loss objective includes economy, reduction of anxiety and maintaining external
obligation.
i. The first objective is that the firm should prepare for potential losses in the most economical
possible way. This involves as analysis of safety program expenses, insurance premiums and the
costs associated with the different techniques of handling losses.
ii. The second objective is the reduction of anxiety. In a firm, certain loss exposures can cause
greater worry and fear for the risk manager. For example, a threat of a lawsuit from a defective
product can cause greater anxiety than a possible small loss from a minor fire. However, the risk
manager wants to minimize the anxiety and fear associated with such loss exposures.
iii. The third pre-loss objective is to meet any externally imposed obligations. This means that
the firm must meet certain obligations imposed on it by the outsiders. For example, government
regulations may require a firm to install safety devices to protect workers from harm. Thus, the
risk manager is expected to see that these externally imposed obligations are met properly.

2) Post-Loss Objectives
Dear learner, the risk management objectives what we have seen so far are objectives which
will be planned to treat risk exposures before its occurrence. The firm or organization has also
risk management objectives to treat losses after their occurrence. This includes the following.

i. Survival: The first and most important post loss objective is survival of the firm: it means
that after a loss the firm can at least start / resume partial operation with in some reasonable
period of time.
ii. Continue operation: for some firms the ability to continue is very important. Especially a
firm such as public utility firm and for firms that may lose customers to competitors if they
cannot operate after occurrence of a loss this objective is indispensable. Example, banks,
bakeries and other competitive firms
iii. Stability of earning is the objective of a firm to maintain earning per share after a loss
occurs. This objective is similar with continue operation objective because earning per share can
be maintained only if the firm continues its operation.

21
iv. Continued growth of the firm: The firm may grow by developing new products and new
market or by acquisitions and mergers products. The risk manager should consider the impact
that a loss will have on the firm’s ability to grow.
v. Finally, the goal of social responsibility will be considered by the risk manger to minimize
the impact that a loss has on other persons and on the society.

 Activity 1

1) List and explain both pre-loss and post loss objectives of risk management program?
______________________________________________________________________________
______________________________________________________________________________
2) If you are assigned as a risk manager for which objectives would you give great emphasis?
_____________________________________________________________________________
1.3 The Risk Management Process

 Do you guess steps to be followed in risk management process?

______________________________________________________________________________

! In order to have successful and effective risk management program a risk manager must

follow certain procedures or steps. The following are the steps that have to be followed so as to
have sound risk management program. Whether the concern is with a business or an individual
situation, the same general steps can be used to analyze systematically and deal with risk. This is
known as Risk Management Process. These four steps to be followed are:
2.3.1 Identification of potential losses
2.3.2 Evaluation of potential losses
2.3.3 Selecting the appropriate techniques of risk management
2.3.4 Implementation and administration of the program.

22
2.3.1 Identification of Potential Losses

 Dear learner, how could you identify the existence of risk in a given firm?

______________________________________________________________________________

! Risk identification is a process by which a business (a risk manger of a business)

systematically and continuously identifies property, liability and personal risk exposures as soon
as or before they emerge.
Unless the risk manager identifies all the potential losses confronting the firm, he/she will not
have a sound opportunity to evaluate it and then determine an appropriate way to handle the
undiscovered risks.
To identify the possible risk exposures the risk manager has to dig and look in to all the
operations of the company. This enables to know where exactly risk emanates from.
Some risks are relatively obvious and some others are hidden in nature, since both obvious and
hidden risks need to be identified the risk manager has to use different sources of information.
Normally, risk manager has several sources of information that can be used to identify the loss
exposures.

! Risk identification is the process by which an organization is able to learn areas in which it is

exposed to risk. Identification techniques are designed to develop information on sources of risk,
hazards, risk factors, perils, and exposures to loss. It seems quite logical to inquire in to the
sources of organizational risks at this particular moment.

A. Sources of Risk
Sources of risk are the sources of factors or hazards that may contribute to positive or negative
outcomes. Sources of risk can be classified in several ways. For instance, the following sources
of risk represent one listing:

23
i. Physical Environment: Clearly, the physical environment is a fundamental source of risk.
Earthquakes, drought, or excessive rainfall can all lead to loss. The ability to fully understand our
environment and the effects we have on it - as well as those it has on us - is a central aspect of
this source of risk. The physical environment may be a source of opportunity as well, for
example, real estate as an investment, agribusiness, and weather as a contributing factor to
tourism.
ii) Social Environment: Changing traditions and values, human behavior, social structures, and
institutions are a second source of risk.
iii) Political Environment: Within a single country, the political environment can be an
important source of risk. In the international realm, the political environment is even more
complex. Not all nations are democratic in their form of government, and some have very
undemocratic attitudes and policies toward business. Foreign assets might be confiscated by a
host government or tax policies might change dramatically. The political environment also can
promote positive opportunities through fiscal and monetary policy, enforcement of laws, and the
education of the population.
iv) Legal Environment: The expected laws and directives may be issued by the government
which may render risky environment to the businesses operating in the country. In the
international domain, complexity increases because legal standards can vary dramatically from
country to country. The legal environment also produces positive outcomes in the sense that
rights are protected and that the legal system provides a stabilizing influence on society.
v) Operational Environment: Processes and procedures of an organization generate risk and
uncertainty. A formal procedure for promoting, hiring, or firing employees may generate a legal
liability. The manufacturing process may put employees at risk of physical harm. Activities of an
organization may result in harm to the environment. International businesses may suffer from
risk or uncertainty due to unreliable transportation systems. The operational environment also
provides gains, as it is the ultimate source of the goods and services by which an organization
succeeds or fails.
vi) Economic Environment: Although the economic environment often flows directly from the
political realm, the dramatic expansion of the global marketplace has created an environment that
is greater than any single government. Although a particular government’s actions may affect

24
international capital markets, control of capital markets is beyond the reach of a single nation.
Inflation, recession, and depression are now elements of interdependent economic systems. On a
local level, interest rates and credit policies can impose significant risk on an organization.
Vii) Cognitive Environment: A risk manager’s ability to understand, see, measure, and assess is
far from perfect. An important source of risk for organizations is the difference between the
perception of the risk manager and reality. The cognitive environment is a challenging source of
risk to identify and analyze. The analyst must contemplate such questions as “How do we
understand the effect of uncertainty on the organization? and “How do we know whether a
perceived risk is real?” An evaluation of the cognitive environment partly addresses the
distinction between risk and uncertainty as defined in Chapter 1.

B. Identification of Exposures
A given peril or hazard can originate in any one of several environments. Fire, for example,
could arise from the physical environment (a lightning strike) or the social environment (arson,
civil unrest). Sources of risk are essentially of no concern to an organization unless that
organization is exposed or vulnerable to the perils that arise from those environments. Therefore,
an important aspect of risk identification is exposure identification. Although in the broadest
sense an entire organization is at exposure to risk, it is useful to develop categories of exposures
for analytical purposes.

! This reading material considers four categories of risk exposures: physical asset exposures,

financial asset exposures, liability exposures, and human exposures.

i. Physical Exposures: Ownership of property gives rise to possible gains or losses to physical
assets and to intangible assets (goodwill, political support, intellectual property), that arise from
these exposures. Property may be damaged, destroyed, lost, or diminished in value in a number
of ways. The inability to use property for a period of time, the so-called time element loss, is
often overlooked by individuals and organizations. Conversely, property exposures to risk may
result in gain or enhancement.

25
ii. Financial Exposures: Ownership of securities such as common stock and mortgages creates
this type of exposure. This exposure can occur either from ownership of the security or when the
organization issues a security held by others. A financial asset conveys rights that are
enumerated in financial terms, such as the right to receive income or the right to purchase an
asset at a specified price. Unlike physical property, loss or gain to a financial asset can occur
without any physical change in the asset itself. Often these gains and losses occur as a
consequence of changing market conditions or changes in the value of the rights conveyed by the
security as perceived by investors.
iii) Liability Exposures: Obligations imposed by the legal system create this type of exposure.
Civil and criminal law detail obligations carried by citizens; state and federal legislatures impose
statutory limitations on activities; governmental agencies promulgate administrative rules and
directives that establish standards of care. Legal obligations that differ from country to country
are an increasingly important aspect of this area.

Unlike property exposures to risk, liability exposures do not have an upside. That is, liability
exposures generally can be considered pure risks. It is true that the law establishes rights as well
as obligations, and the enforcement of a right can result in a gain.

iv) Human Exposures: Part of the wealth of an organization arises from its investment in
humans: the human resources of the organization. Possible injury or death of managers,
employees, or other significant stakeholders (customers, Secured creditors, stockholders, and
suppliers) exemplifies this type of exposure. Human asset exposures also can lead to gains, as
exemplified by improvements in productivity. One might, for example, view a highly technical
piece of machinery as source of loss (worker injury) and gain (increased productivity). In such a
case, the risk management strategy is likely to incorporate elements that will reduce the potential
for loss while maximizing the likelihood of gain (employee training, for instance). As a final
note, loss of human assets does not always imply physical harm. Economic insecurity is a
common type of loss, unemployment and retirement being excellent examples. Both the physical
and economic welfare of human beings are components of this type of exposure to risk.

26
C. The Range of Risk Identification Techniques

 How is risk to be identified?

______________________________________________________________________________
_______________________________________________________________
Where would you begin to start the task of identifying risk in a major factory complex, a
shopping center, an airport, a department store chain, a bank? Do you expect that you would
arrive at the premises, assuming always that the actual premises existed and that we were not
concerned with risks at the planning stage, with a clipboard to begin the task? The world of
industry and commerce is far too complex and sophisticated to allow for proper risk identifica-
tion simply by a ‘walk round the premises’.

Specific techniques will have to be employed to aid your identification of risk. However, no one
method for risk identification will be appropriate for all forms of risk, or even for similar forms
of risk in different situations.

! Risk identification techniques include:

1) Organizational Charts
Organizational charts are intended to highlight broad areas of risk rather than specific, individual
risks such as fire, security or liability. The organizational chart encourages the risk identifier to
take a birds-eye view of the organization: to stand back and above the day-to-day operation and
take stock of the risks which exist. This term ‘risk identifier’ does need some explanation. In
many organizations there will be a risk or insurance manager employed whose job, in part, will
be the identification of risk. Where no risk manager exists, it may be that the insurance company
performs the risk identification function. In other cases, an insurance broker or consultant may
take on the role of identifying risk. The term risk identifier is intended to refer to anyone who
has the task of identifying risk.

27
Most organizations will have charts of some kind or another. Even if they are only in publicity
material, there will be some starting point for building a suitable organizational chart. It is wise
to involve as many people as possible in the construction of the chart, in order to ensure that it is
not unrealistic or over-simplistic in its make-up.

2) Physical Inspections/onsite inspections


The organizational chart took a very broad view of the risks to which an organization could
expose. The physical inspection of premises, plant or processes takes a different approach.
Everyone understands what is meant by physical inspection, it is possibly the most common and
best understood of all the techniques available. On-site inspections are a must for the risk
manager. By observing firsthand the organization’s facilities and the operations conducted
thereon the risk manager can learn much about the exposures faced by the firm.

3) Checklists
Checklists deal with the particular problem of the time-consuming nature of physical inspections.
The basic idea of the checklist is that a pro-forma is sent to the site for completion by someone
there. This dispenses with the need for a physical inspection and hence cuts the time and cost of
identification.

The checklist acts as the source of information about risk. It really takes the place of the personal
visit and so it has to be drawn up very carefully. It is wise, when constructing a checklist for the
first time, to consult as widely as possible in order to ensure that all aspects of risks are taken
into account. In particular, the following points are worth keeping in mind:

 The checklist should be simple to understand


 The checklist should be free from ambiguity.
 The checklist should be short
 The checklist should not be threatening

28
4) Flow charts

! Flow charts are schematic representations of a sequential process. A flow chart depicting the

operations of a firm can guide a risk manager to risks associated with those operations.
In many organizations there is some kind of flow. This could take the form of:
Production flow, where raw materials come in at one end of a process and a finished product
emerges at the other end. There was an identifiable flow through the system.
Service flow, where there may not be raw materials but the business may depend on flow of
another form. It could be the flow of people, as in the case of a restaurant or hotel.
Money flow, as in the case of a bank or an insurance company. Money comes in at one end and
various promises are made, the effects of which are seen at some later date.
In each case there are various stages in the flow, and at each stage there are risks which could
impede or halt the flow. Any interruption to the flow will have consequences for revenue and
profit. A flow chart can be used to identify the key stages, and structure the analysis of the risks
at each stage.
5) The Financial Statement Method
The financial statement method was proposed by A.H. Criddle (1962). Although this approach
was intended for private organizations, the concepts of the financial statement approach can be
generalized in public sector organizations as well. By analyzing the balance sheet, operating
statements, and supporting documents, Criddle maintains, the risk manager can identify
property, liability, and human asset exposures of the organization. By coupling these statements
with financial forecasts and budgets, the risk manager can discover future exposures. Financial
statements reveal this information because every organizational transaction ultimately involves
either money or property.
Under this method, each account title is studied to determine what potential risks it creates. The
results of the study are reported under the account titles. Criddle argues that this approach is
reliable, objective, based on readily available data, presentable in clear, concise terms, and able
to be applied by either risk managers or professional consultants. Moreover, it translates risk
identification into financial terminology familiar to other managers, accountants, and bankers.

29
6) Interactions with Other Departments
Frequent interactions with other departments provide another source of information on exposures
to risk. These interactions may include oral or written reports from other departments on their
own initiative or in response to a regular reporting system that keeps the risk manager informed
of developments. The importance of such a communications network should not be
underestimated. These departments are constantly creating or becoming aware of exposures that
might otherwise escape the risk manager’s attention. Indeed, the risk manager’s success in risk
identification is heavily dependent on the cooperation of other departments.

7) Interactions with Outside Suppliers and Professional Organizations


In addition to communicating with other departments, the risk manager normally interacts with
outsiders who provide services to the organization. These outsiders, for example, might include
accountants, lawyers, risk management consultants, actuaries or loss-control specialists. The
objective would be to determine whether the outsiders have identified exposures that otherwise
would be missed. Possibly, the outsiders themselves may create new exposures.

Involvement with professional organizations and use of published material is another valuable
source of information. For example, the annual meeting of the Risk and Insurance Management
Society normally includes sessions focusing on specific problems faced by areas of industry. In
addition, a number of organizations that focus on specialized areas of risk management have
been formed in response to demands of risk managers in these areas.
8) Contract Analysis
Many of an organization’s exposures to risk arise from contractual relationships with other
persons and organizations. An examination of these contracts may reveal areas of exposures that
are not evident from the organization’s operations and activities. In some cases, contracts may
shift responsibility to other parties.
9) Statistical Records of Losses
Where statistical records of losses are available, they can be used to identify sources of risk.
These records may be available from risk management information systems developed by
consultants or, in some cases, the risk manager. These systems allow losses to be analyzed
according to cause, location, amount, and other issues of interest.

30
10) Incident Reports
A network of information sources can be very useful in identifying possible losses. Ideally, the
information provided through this network should include not only reports of accidents and near
accidents, but also reports of incidents that could have resulted in injury or damage but
presumably did not. Frequently, good fortune and luck allow a person to escape without injury
from an incident that posed a serious threat. Information on these events is useful in preventing
injury or damage if the circumstances are repeated, but only if the risk manager is aware of the
potential problem. A system for reporting of incidents usually includes a form for recording
important information. In addition to date, time, location and identity of parties involved in the
incident, the form should request information that later could prove helpful in preventing similar
occurrences or mitigating the injury or damage if it occurs.

D. Common Features of Risk Identification


We have looked at a number of individual techniques and it is now necessary to say something
of a more general nature about the task of risk identification, regardless of the technique selected.
We can do this by commenting on a number of common features of risk identification.

The task of risk identification must be given the proper priority in an organization. It’s an im-
portant function, as many of the risks which are to be identified can put their way into the very
core of the existence of the organization itself.
There is a range of techniques available and no one technique can be used in all situations. As we
have dealt with each technique, we have commented on the relevant uses to which it can be put.
Thought must be given to the nature of each risk and the best technique, or combination of
techniques, selected.
The task of risk identification is a continuing one: the one-off exercise is of little value in many
practical cases. The nature of industry is such that it is constantly changing and it is therefore
essential that risk identification takes place at regular intervals.
Efficient record keeping is an important part of identification of risk. A great deal of valuable
information is obtained at the time of risk identification, and this should be stored carefully for
later use and referral.
Other people, in addition to the risk identifier, should be involved in the process of risk identi-

31
fication whenever possible. Organizations are complex and no one person will have all the
knowledge which will be required to enable risks to be identified.
The cost of risk identification must be remembered. There is little point in spending Br.10 to
identify risks which in the worst case can only ever cost Br. 1. Identifying risk is important, but it
costs money and this cannot be overlooked. Finally, a measure of common sense and imagi-
nation are valuable attributes to have when flying to identify risk
2.3.2. Evaluating/Measuring Potential Losses

 Dear learner, do you know how the identified risk is measured?

! After, risk manager has identified the various types of potential losses faced by his or her

firm. These exposures must be measured in order;


1) To determine their relative importance
2) To obtain information that will help the risk manager o decide up on the most desirable
combination of risk management tools.
Evaluating and measuring the impact of losses on the firm involves an estimation of the potential
frequency and severity of loss. Information is needed concerning two dimensions of each
exposure:
1) The loss frequency refers to the probable number of losses that may occur during some given
time period.
2) The loss severity refers to the probable size of the losses that may occur.
The risk manager, by using available data from past experience, tries to construct a probability
distribution of the number of events and/or the probability distribution of total monetary losses.
This would, indeed, require knowledge of certain statistical techniques and concepts (You can
refer your business statistics). The probability distribution of number of events and/or total
monetary losses would enable the risk manager to estimate, among other things, the size of
possible monetary losses and the corresponding probabilities of occurrence.

32
 Activity 2

1) What are the various approaches that may be used by the risk manager in the identification of
loss exposure?
______________________________________________________________________________
2) What is loss frequency and loss severity, for which you give more emphasis?
2.3.3 Selecting the Appropriate Techniques of Risk Management

 Dear learner, could you guess the techniques used to manage/handle risk?

______________________________________________________________________________

! After the risk manager has identified and measured the risks facing the firm, he or she must

decide to handle them. There are two basic approaches. First, the risk manger can use risk
control measures to alter the exposures in such a way as (1) to reduce the firm’s expected
property, liability, and personnel losses, or (2) to make the annual loss experience more
predictable.
First, Risk control measures include avoidance, loss prevention, separation/diversification,
combination, and some transfers.

Second, the risk manger can use risk-financing measures to finance the losses that do occur.
Funds may be required to repair or restore damaged property, to settle liability claims, or to
replace the services of disabled or deceased employees or owners. In some cases, the firm will
decide not to restore the damaged property or replace the disabled or decreased person.
Nevertheless, it may also have suffered a financial loss through a reduction in its assets or its
future earning power. The tools in this second category include retention/assumption, self
insurance, noninsurance transfers, and insurance.
A. Risk Control Techniques: attempt to reduce the frequency and severity of accidental losses
to the firm. These include the following:

33
i. Risk Avoidance
One way to control a particular pure risk is to avoid the property, person, or activity with which
the exposure is associated by 1) refusing to assume it even momentarily or 2) an exposure
assumed earlier, most examples of risk avoidance fall in the risk category. To illustrate a firm
can avoid a flood loss by not building a plant in a flood plain. An existing loss exposure may also
be abandoned.
For example, a firm that produces a highly toxic product may stop manufacturing that product.
Similarly, an individual can avoid third party liability by not owning a car. Product liability can
be avoided by dropping the product. Leasing avoids the risk originating from property
ownership.

The major advantage of avoidance is that the chance of loss is reduced to zero if the loss
exposure is not acquired. In addition, if an existing loss exposure is abandoned, the possibility of
loss is either eliminated or reduced because the activity or product that could produce a loss has
been abandoned.

Avoidance, however, has two disadvantages. First, it may not be possible to avoid all losses. For
example, a company cannot avoid the premature death of a key executive. Similarly, a business
has to own vehicles, building, machinery, inventory, etc… Without them operations would
become impossible. Under such circumstances avoidance is impossible. In fact there are
circumstances where avoidance is a viable alternative. For example, it may be better to avoid
the construction of a company near river bank, volcano-prone areas, valleys, etc. because the risk
is so great.
The second disadvantage of avoidance is that it may not be practical or feasible to avoid the
exposure. For example, a paint factory can avoid losses arising from the production of paint.
However, without any paint production, the firm will not be in business.

ii) Loss Prevention and Reduction


These measures refer to the safety actions taken by the firm to prevent the occurrence of a loss
or reduce its severity if the loss has already occurred. Prevention measures, in some cases,
eliminate the loss totally, although their major effect is to reduce the probability of loss

34
substantially. Loss reduction measures try to minimize the severity of the loss once the peril
happened. For example, auto accidents can be prevented or reduced by having good roads, better
lights and sound traffic regulation and control, fast first-aid service and control, fast first-aid
service and the like. Loss prevention and retention measures must be considered before the risk
manager considers the application of any risk financing measures.

 Dear learner, what measures do you take to control risks that you may face?

_____________________________________________________________________________

! Following are some of the loss prevention measures:

 Research on fire protection equipment and appliances.

 Construction using fire insensitive materials.

 Automatic smoke detectors, fire alarms.

 Burglar alarms in costly business situation, jewelry, and diamonds.

 Location choice, avoiding construction near petrol stations, chemical reservoirs, waste
disposal areas, etc.

 Tight quality control to prevent risk of product liability.

 Educational programs to the public using available media.

 Multiple suppliers, buffer stocks.

 Safety measures, adequate lighting, ventilation, special work clothes to prevent industrial
accidents.

 Regular inspection of machinery to prevent explosions, breakdowns, etc..

 Accounting controls (Internal Control).

35
 Electronic metal detectors to check passengers for arms and explosives in the airline
business.

 Automatic gates at crossing lines to prevent collisions train and motor vehicles.

 Warning posters (No smoking!! Danger zone!!)


Loss Reduction Measures:

 Installing automatic sprinklers.

 First aid kit

 Evacuation of people,

 Immediate clean-up operations,

 Fire extinguishers, guards.


Appropriate measures take to prevent accidents bring benefits not only to the firm, but also to the
society as well. For example, a destruction of inventory of a firm could be a total loss to the firm
in particular. The society also faces a real economic loss because those goods are no more
available to people. Thus, the importance of Loss Prevention and Reduction measures should not
be underestimated by a firm. To design effective loss prevention and reduction measures, it may
be helpful to identify the causes of accidents.

! Some of the causes of accident and the possible loss prevention and reduction measures are

indicated below.

Causes of accidents Loss prevention measures


1. Working on dangerous equipment with less 1. Safety seminars, inspection at regular
care times
2. Improper use of equipment 2. Training, safety seminars
3.Violating Safety Procedures and Regulations. 3. Safety seminars, warning, dismissal
4. Human error, Negligence 4.Training, safety seminars
5. Use of inappropriate tools 5. Provide appropriate tools

36
6. Lack of protective clothing 6. Provide necessary protective clothing
7. Use of defective equipment 7.Regular inspection and maintenance
8. Inadequate Knowledge about the job 8. Training
9. Working while physically ill 9. Sick leave, don’t allow to work until
recovery
10. Mental Disturbance of employee 10. Day-off to the employee

Date should be kept regarding accidents occurred. The causes of these accidents must be
investigated. Pre-designed forms may be employed to report on accidents and their causes. This
could allow for the design of a much better Loss Prevention and Reduction measures.

Loss Prevention and Reduction measures entail costs. These costs include expenditures for the
acquisition of safety equipment and services, operating expenses such as salary payments to
guards, inspectors, safety engineers and other employees engaged in safety work. Other costs are
also incurred in connection with safety training and seminars. The risk manager will have to
design the Loss Prevention and Reduction measures in the most efficient way in order to
minimize such costs without reducing the desired safety level.

iii. Risk Separation/Diversification

 Dear learner, what do you think separation mean as risk controlling techniques?

____________________________________________________________________________
Don’t put all your money in one basket!
The above axiom shows the importance of separation in risk management. Therefore,
Separation of the firm’s exposures to loss instead of concentrating them at one location where
they might all be involved in the same loss is the third risk control tool. For example, instead of
placing its entire inventory in one warehouse the firm may elect to separate this exposure by
placing equal parts of the inventory in four widely separated warehouse; one in Adama, One in
Mekelle, the other in Diredawa, and the other in Harar. To the extent that this separation of
exposures reduces the maximum probable loss to one event, it may be regarded as a form of loss
reduction. Emphasis is placed here, however, on the fact that through this separation the firm

37
increases the number of independent exposure units under its control. Other things being equal,
because of the law of large number, this increase reduces the risk, thus improving the firm’s
ability to predict what its loss experience will be.
iv) Combination/Pooling

 Dear student, what do you think combination means as risk controlling technique

______________________________________________

! Combination is a basic principle of insurance that follows the law of large numbers.

Combination increases the number of exposure units since it is a pooling process. It reduces risk
by making loses more predictable with a higher degree of accuracy. The difference is that unlike
separation, which spreads a specified number of exposure units, combination increases the
number of exposure units under the control of the firm.

In the case of firms, combination results in the pooling of resources of two or more firms. One
way a firm can combine risks is to expand through internal growth. For example, a taxi-cab
company may increase its fleet of automobiles. Combination also occurs when two firms merge
or one acquires another. The new firm has more buildings, more automobiles, and more
employees than either of the original companies. This leads to financial strength, thereby
minimizing the adverse effect of the potential loss. For example, a merger in the same or
different lines of business increases the available resources to meet the probable loss.

B. Risk Finance Techniques

 As you think, what is a risk financing technique? And what makes it differ from risk

controlling techniques?
______________________________________________________________________________
______________________________________________________________________________

38
Risk financing techniques provide for the funding of accidental losses after they occur. These
include:
i) Retention/Assumption
Retention means that the firm retains part or all of the losses that result from a given loss
exposure. Retention can be effectively used in a risk management program when certain
conditions exist.
First, no other method of treatment is available. Insurers may be unwilling to write a certain type
of coverage, or the coverage may be too expensive. Noninsurance transfers may not be available.
Loss control can reduce the frequency of loss, but not all losses can be eliminated. In these cases,
retention is a residual method. If the exposure cannot be insured or transferred, then it must be
retained.

Second, the worst possible loss is not serious. For example, physical damage losses to
automobiles in a large firm's fleet will not bankrupt the firm if the automobiles are separated by
wide distances and are not likely to be simultaneously damaged.

Finally, losses are highly predictable. Retention can be effectively used for workers'
compensation claims, physical damage losses to automobiles, and shoplifting losses. Based on
past experience, the risk manager can estimate a probable range of frequency and severity of
actual losses. If most losses fall within that range, they can be budgeted out of the firm's income.

ii. Self Insurance

! Self –insurance is a special from of planned retention by which part or all of a given loss

exposure is retained by the firm. A better name for self insurance is self-funding, which
expresses more clearly the idea that losses are funded and paid by the firm.
Risk retention, planned or unplanned, should not be confused with the concept of self-insurance.
Although self insurance requires risk retention, it implies an attempt by a business to combine a
sufficient number of its own similar exposures to predict the losses accurately. Furthermore, a
self-insurance plan implies that adequate financial arrangements have been made in advance to

39
provide funds to pay for losses should they occur. Unless payments to the self-insurance fund are
calculated scientifically and paid regularly, a true self-insurance system does not exist.

Self insurance plan are distinguished from other insurance operations by having the pooling of
exposures and funding of the cost of losses take place within one business entity.

iii. Non-insurance Transfer


Non-insurance Transfers is another method of handling losses. Non-insurance transfers are
methods other than insurance by which a pure risk and its potential financial consequences
are transferred to another party. Examples of non-insurance transfers include contracts, leases
and hold-harmless agreements.
For example, a company’s contract with a construction firm to build a new plant can specify that
the construction firm is responsible for any damage to the plant which it is being built.
A firm’s computer lease can specify that maintenance, repairs and any physical damage loss to
the computer are the responsibility of the computer firm. Otherwise, a firm may insert a hold-
harmless clause in a contract, by which one party assumes legal liability on behalf of another
party. Thus, a publishing firm may insert a hold-harmless clause in a contract, by which the
author and not the publisher is held legally liable if anybody sued the publisher.
iv) Insurance

Commercial insurance can also be used in a risk management program. Insurance can be
advantageously used for the treatment of loss exposures that have a low probability of loss but
the severity of a potential loss is high.

! If the risk manager decides to use insurance to treat certain loss exposures, five key areas

must be emphasized.

1) Selection of insurance coverage’s

2) Selection of an insurer (insurance company)

3) Negotiation of terms

40
4) Dissemination of information concerning insurance coverage

5) Periodic review of the insurance programs

1) Selection of insurance coverage’s


The risk manager must select the insurance coverage’s needed. Since there may not be enough
money in the risk management budget to insure all possible losses, the need for insurance can be
divided into three categories;
i) Essential Insurance includes that coverage’s required by law or by contract, such as workers
compensation insurance. It also includes that coverage’s that will protect the firm against a loss
that threatens the firm’s survival.
ii) Desirable Insurance is protection against losses that may cause the firm financial difficulty,
but not bankruptcy
iii) Available insurance is coverage for slight losses that would merely inconvenience the firm.
2) Selection of an Insurer
The next step is that the risk manager must select an insurer or several insurers. Here, several
important factors are to be considered by the risk manager. These include the following:
 Financial strength of the insurer
 Risk management services provided by the insurer
 The cost and terms of pro
3) Negotiation of terms
After the insurer is selected, the terms of the insurance contract must be negotiated. If printed
policies, endorsements and forms all used, the risk manager and insurer must agree on the
documents that will form the basis of the contract. If a specially tailored manuscript policy is
written for the firm, the language and meaning of the contractual provisions must be clear to both
parties. If the firm is large, the premiums are negotiable between the firm and insurer.
4) Dissemination of information concerning insurance coverage
Information concerning insurance coverage must be given to others in the firm. The firm’s
employees must be informed about the insurance coverage, the records that must be kept, the
risk management services that the insurer will provide, etc.

41
5) Periodic review of the insurance program

The entire process of obtaining insurance must be evaluated periodically. This involves an
analysis of agent and broker relationships, coverage needed, cost of insurance, quality of loss-
control services provided, whether claims are paid promptly, etc

Selecting the Most Appropriate

 Dear Learner, which method should be used in risk management?

______________________________________________________________________________

In determining the appropriate method or methods of handling losses, the below matrix can be
used. It classifies the various loss exposures according to frequency and severity.

Risk Management Matrix

Type of Loss Frequency Loss Severity Appropriate Risk Management


Loss Technique
1 Low Low Retention

2 High Low Loss Control & Retention

3 Low High Insurance

4 High High Avoidance

Figure 2.1. Risk Management Matrix


The first loss exposure is characterized by both low frequency and low severity of loss. One
example of this type of exposure would be the potential theft of an Office Secretary’s Note pad.
This type of exposure can be best handled by retention, since the loss occurs infrequently and
when it occurs it does not cause financial harm.

42
The second type of exposure is more serious. Losses occur frequently, but severity is relatively
low. Examples of this type of exposure include physical damage losses to automobiles,
shoplifting and food spoilage. Loss control should be used here to reduce the frequency of losses.
In addition, since losses occur regularly and are predictable, the retention technique can also be
used.

The third type of exposure can be met by insurance. Insurance is best suited for low frequency,
high severity losses. High severity means that a catastrophic potential is present, while a low
probability of loss indicates that the purchase of insurance is economically feasible. Examples
include fires, explosion and other natural disasters. Here, the risk manager could also use a
combination of retention and insurance to deal with these exposures.

The fourth and most serious type of exposure is characterized by both high frequency and high
severity. This type of risk exposure is best handled by avoidance. For example, if a person has
drunken and if he attempts to drive home in that drunken stage, the chance of meeting with an
accident is more. This loss exposure can be avoided by not driving at the drunken stage or by
having a driver to drive his car.

2.3.4. Implementing and Administering the Risk Management Program

 Dear learner, can you guess how the final steps of risk management administered?

____________________________________________________________________________
The next and the final step in the risk management process is implementation and administration
of the risk management program. It involves three important components;
(i) Risk management policy statement
(ii) Co-operation with other departments
(iii) Periodic review and evaluation

i) Risk management policy statement


A risk management policy statement is necessary in order to have an effective risk management
program. This statement outlines the risk management objectives of the firm, as well as

43
company policy with respect to the treatment of loss exposures. It also educates top level
executives in regard to the risk management process and gives the risk manager greater authority
in the firm.
In addition, a risk management manual may be developed and used in the program. The manual
describes the risk management program of the firm and can be a very useful tool for training
new employees who will be participating in the program.

ii) Co-operation with other departments


The risk manager has to work in co-operation with other functional departments in the firm. It
will facilitate to identify pure loss exposures and methods of treating these exposures.
The Accounting Department can adopt Internal Accounting Controls to reduce employees'
fraud and theft of cash.
The Finance Department can provide information showing how losses can disrupt profits and
cash flow.
The Marketing Department can prevent liability suits by ensuring accurate packaging. Besides,
safe distribution procedures can prevent accidents.
The Production Department has to ensure quality control and effective safety programs in the
plant can reduce injuries and accidents.
The Personnel Department may be responsible for employee benefit program, pension program
and safety program.

iii) Periodic review and evaluation


The risk management program must be periodically reviewed and evaluated to see whether the
objectives are being attained or not. Especially, risk management costs, safety programs and
loss preventive programs must be carefully monitored. Loss records must also be examined to
detect any changes in frequency and severity. Finally, the risk manager must determine whether
the firm’s overall risk management policies are being carried out, and whether the risk manager
is receiving the total co-operation of the other departments in carrying out the risk management
functions.

44
 Activity 3

1) As a risk manager argues that since avoidance always involves forgoing some activity, it is
undesirable for affirm and society.
a) How would you answer this?
______________________________________________________________________________
b) List three situations in which avoidance might be desirable
______________________________________________________________________________
_________________________________________________________________________
2) List some examples of loss control and non-insurance transfer?
______________________________________________________________________________
____________________________________________________________________
3) Is self insurance the same as insurance? Explain
______________________________________________________________________________
______________________________________________________________________________
__________________________________________________________________________
4) Risk implementation and administration program include three important components. List
and explain them
______________________________________________________________________________
______________________________________________________________________________
_______________________________________________________________________

 Check Lists

Dear learner, below are the most important points which are drawn from the chapter you have
been studying up to now. Put a tick () mark in the box after the point you feel you have
understood. But if you find a concept that you have not mastered yet, please go back and read the
chapter you passed through.

45
I can:

1) Define risk management

2) List objectives of risk management

3) List steps to be followed in risk management

4) Explain risk controlling techniques and identify risk control techniques

5) Explain risk financing techniques and identify risk financing techniques

6) Identify which method to be used

7) Implement and administer risk management program

Chapter Summary
Risk Management is defined as a systematic process for identification and evaluation of pure loss
exposures faced by an organization or individual, and for the selection and implementation of
most appropriate techniques for treating such exposures. It is a science approach to dealing with
pure risks by anticipating possible accidental losses and designing and implementing procedures
that minimize the occurrence of loss or the financial impact of the losses that do occur.

Risk management has several important objectives. Pre-loss objectives include the goals of
economy, reduction of anxiety, and meeting externally imposed obligations. Post loss objectives
include survival of the firm, continued operation, stability of earnings, continued growth, and
social responsibility.

There are four steps in the risk management process. Potential losses must be identified. The
potential losses must then be evaluated in terms of loss frequency and loss severity. An
appropriate method or combination of methods for treating loss exposures must be selected. The
risk management program must be implemented and properly administered.

The major methods for treating loss exposures in a risk management program are classified as
risk controlling and risk financing techniques. The former include avoidance, loss control and

46
reduction, separation or diversification, and combination and the latter include retention, self
insurance, non insurance transfers, and insurance.

A risk management program must be properly implemented and administered. This involves
preparation of a risk management policy statement, close cooperation with other individuals and
departments, and periodic review of the entire risk management program.

 Self Test Questions

Part-I: Say “True” for the correct statement and “False” for the incorrect
statement.
1) Risk is unavoidable and unmanageable.
2) Risk controlling techniques include retention, self insurance, non-insurance transfer, and
insurance.
3) Avoidance is the appropriate risk controlling technique when both frequency and severity of
loss are high.
4) Non-insurance transfers are methods other than insurance by which a pure risk and its
potential financial consequences are transferred to another party.
5) Survival is considered as pre-loss objective of risk management.
6) Law large number states that as a number of exposures unit increases its objective risk also
increase.
7) Ownership of securities such as common stock and mortgages creates physical type of
exposure.
8) Insurance is the appropriate technique of risk financing when the frequency of loss is high but
its severity is low.
9) Combination or pooling makes loss experience more predictable by increasing the number of
exposure units.
10) Risk manager must interact with other department in order to identify risk exposures.

47
Part-II: Choose the best answer among the given alternatives
1) Pre-loss objectives of risk management include all of the following, except?
A. Reduction in anxiety C. Earning stability
B. meeting external imposed obligation D. Economy
2) Retention/Assumption in risk handling technique is advisable when, ______
A. No other method of treatment is available
B. The worst possible loss is serious
C. The losses are less predictable
D. All
3) Which of the following tools is/are enabling us to identify the potential loss in our company?
A. Checklist D. Onsite inspection
B. Flow charts E. All of the above
C. Financial statement
4) Which of the following is/are incorrect concerning risk management?
A. Risk management and insurance management are one and the same
B. Risk measurement is conducted in order to determine the relative importance of loss and to
select appropriate risk management tools
C. For risk controlling and financing techniques we use the same tools.
D. A and C
E. B and C
5) ___________is need for insurance in which coverage for slight losses that would merely
inconvenience to the firm.
A. essential insurance
B. Important insurance
C. Optional insurance
D. Desirable insurance
E. None

48
Part-III: Short Answer Questions
1) List and explain pre-loss and post loss objective of risk management

2) Identify the four categories of risk exposures

3) List the four steps used in risk management process

4) What are several important factors are to be considered by the risk manager in selection of
insurer?

5) What distinguish Self insurance plan from other insurance op

49
CHAPTER THREE
INSURANCE
Introduction

Dear learner, in the first two units you have learned all about what risk and risk management
is. Specially, in the second chapter you have seen risk management process and had an
opportunity to appreciate the different tools of risk management. Among the tools we have
seen insurance as one and well pronounced tool of risk management. In this and the
consecutive chapters of this course we will exclusively deal with insurance. In this unit will
be devoted to help you learn more about modern commercial insurance and try to appreciate
the basic issues of insurance like meaning, function, and basic characteristics, elements of
insurable risk and social cost and benefits of insurance. Insurance plays an extremely
important part in ensuring the economic well being of the country, but it does not have a high
profile and therefore many people have little idea of the full role it plays.

Objectives

After studying this chapter, you should be able to:


 Define insurance and explain the basic characteristics of insurance
 Identify the characteristics of insurable risks
 Show how insurance differs from gambling and speculation
 List the requirements of an insurable contract
 Identify and explain the costs and benefits of insurance

3.1. Insurance Defined

 Dear learner, would you define insurance?

_____________________________________________________________________________

! Insurance can be defined in several ways and probably no one brief definition does fit to

its many new features. It may be defined from economic, legal, business, social and

50
mathematical point of views.
In economic sense, for instance, insurance is a mechanism of providing certainty or
predictability of loss with regard to pure risk. It accomplishes these by policy or charity of
risk. By reducing uncertainty in the business environment, it will create peace of mind that
enables businesspersons focus on their primary activities instead of worrying about the
existence of possibility of loss so that societies can grow more.

From legal point of view, insurance is a contract whereby, a consideration (price) paid to a
party adequate to the risk, becomes security to the other that he/she shall not suffer loss,
damage or prejudice by the happening of risks specified in the contract for which he/she may
be exposed to. The contracting parties are the insured, who is responsible to pay the price for
obtaining the security (premium), and the insurer, who will assume the risk transferred. This
makes insurance a means of transferring risk for a premium (price) from one party known as
the insured to another called insurer.

From business perspective insurance is defined as a cooperative device to spread the loss
caused by a particular risk over a number of persons who are exposed to and who agree to
ensure themselves against that risk. Every risk involves the loss of one or other kind. The
function of insurance is to spread the loss over a large number of persons who agreed to
cooperate each other at the time of loss. The risk cannot be averted but loss occurring due to a
certain peril can be distributed amongst the agreed persons. They agree to share the loss
because the chance of loss, i.e., the time and amount, to a person is not known.

Any of the insured's may suffer loss to a given risk; so, the rest of the persons who have
agreed will share the loss. The larger number of such persons insured, the easier the process
of distribution loss. In fact, they share the loss by payment of premium, which is calculated
on the basis of probability of loss.

From the social point of view insurance is defined as a device to accumulate funds to meet
uncertain losses of capital, which is carried at through the transfer of the risk of many
individual to one person or, to a group of persons.

Mathematically, insurance is the application of certain actuarial principles (insurance


mathematics). Law of probability and statistical techniques are used to achieve predictability.

51
In general insurance is an economic system for reducing uncertainty of loss through pooling
of losses together, a legal method of transferring risk from the insured to the insurer in a
contract of indemnity, a business undertaking for profit that provides many jobs in a free
enterprise economy, a social device in which the loss of few is covered by the contribution of
many, or an actuarial system of applied mathematics.

 Activity 1

Define Insurance using your own word


___________________________________________________________________________

The Functions of Insurance

! The functions of insurance can be studied in two parts: primary and secondary

functions.
1) Primary Functions
Insurance executes the following functions primarily.
a) Providing certainty: Insurance provides certainty of payment at the uncertainty of loss. The
uncertainty of loss can be reduced by better planning and administration. Insurance removes
all uncertainties and assurance is given to payment of compensation at the time of loss. The
insurer charges premium for providing the said certainty.
b) Protection: The main function of insurance is to provide protection against the probable
chances of loss. Insurance guarantees the payment of loss and this protects the assured from
sufferings.
c) Risk-sharing: When the risk takes place, all the persons who are exposed to the risk share
the loss.

2) Secondary Functions
In addition to the aforementioned primary functions, insurance plays the following:
a) Prevention of loss: Insurance is primarily concerned with the financial consequences of
losses, but it would be fair to say that insurers have more than a passing interest in loss
control. It could be argued that insurers have no real interest in the complete control of loss,
as this would inevitably lead to an end to their business. This is a rather shortsighted view.

52
Insurers do have an interest in reducing the frequency and the severity of loss. In a practical
way, buyers of insurance will normally come into contact with the loss control services
offered by an insurer when they meet the surveyor. The surveyor may be employed by the
insurer, or indeed the insurance broker, and part of his/her job is to give advice on loss
control. Many insurers employ specialist surveyors in fire, security, liability and other types
of risk; others will employ people with broader, but less detailed, knowledge.

b) Providing Capital: Insurance companies have, at their disposal, large amounts of money.
This arises due to the fact that there is a time gap between the receipt of a premium and the
payment of a claim. A premium could be paid in January and a claim may not occur until
December, if it occurs at all. The insurer has this money and can invest it. In fact, the insurer
will have the accumulated premiums of all insured’s, over a long period of time.

We have listed investment as one of the benefits of insurance in later discussions and the
benefit lies in the use to which the money is put. Insurers invest in a wide range of different
forms of investment. By having spread of investments, the insurance industry helps national
and international businesses in their borrowing. It also helps industry and commerce, by
making various forms of loan and by taking up shares which are offered on the open market.
Insurers make up part of what are termed the institutional investors; the others include banks,
building

3.2. Basic Characteristics of Insurance

 Dear learner, would you guess the basic characteristics of insurance?

___________________________________________________________________________

! An insurance plan or arrangement typically has certain characteristics. They include the

following:
1) Pooling of losses
2) Payment of fortuitous losses
3) Risk transfer
4) Indemnification

53
1) Pooling of Losses
Pooling of losses is the heart of insurance. The other names for pooling are sharing, spreading
or combination. "Pooling is the spreading of losses incurred by the few over the entire group,
so that in the process, average loss is substituted for actual loss". In addition, pooling
involves the grouping of a large number of homogeneous exposure units so that the law of
large numbers can operate to provide a substantially accurate prediction of future losses.
Homogeneous exposure unit means there is a large number of similar (e.g., houses), but not
necessarily identical exposure units that are exposed to the same perils. Thus pooling implies:
 The sharing of losses by the entire group and
 The prediction of future losses with some accuracy based on the law of large
numbers.

2) Payment of Fortuitous Losses


A second characteristic of private insurance is the payment of fortuitous losses. A fortuitous
loss is one that is unforeseen and unexpected and occurs as a result of chance. In other words,
the loss must be accidental. The law of large numbers is based on the assumption that losses
are accidental and occur randomly.

3) Risk Transfer
Risk transfer is another essential element of insurance. With the exception of self- insurance,
a true insurance plan always involves risk transfer. Risk transfer means that a pure risk is
transferred from the insured to the insurer, who typically is in a stronger financial position to
pay the loss than the insured. From the view point of the individual, pure risks that are
typically transferred to insurers include the risk of premature death, poor health, disability,
destruction and theft of property, and liability lawsuits.

4) Indemnification
A final characteristic of insurance is indemnification for losses. Indemnification means that
the insured is resorted to his or her approximate financial position prior to the occurrence of
the loss. Thus, if your house burns in a fire, the homeowner’s policy will indemnify you or
restore you to your previous position. If you are sued because of the negligent operation of an
automobile, your automobile liability insurance policy will pay those sums that you are
legally obligated to pay.

54
3.3. Fundamentals of Insurable Risk

 Dear learner, what do think a given risk should fulfill to be insurable?

___________________________________________________________________________
As we have seen in the second chapter risk management deals with pure risk exposure and
consequently insurance normally insure/deals only with pure risk exposures. However, not
all pure risks are insurable. Certain requirements usually must be fulfilled before a pure risk
can be privately insured. From the view point of the insurer, there are ideally six
requirements of an insurable risk.
1. There must be a large number of homogeneous exposure units
2. The loss must be accidental and unintentional
3. The loss must be determinable and measurable.
4. The loss should not the catastrophic
5. The chance of loss must be calculable
6. The premium must be economically feasible
1) There must be a large number of homogeneous exposure units
The purpose of the first requirement is to enable the insurers to predict losses based on the
law of large numbers. If a sufficiently large number of homogeneous exposure units are
present within a class, the insurer can accurately predict both the average frequency and the
average severity of loss. The items in an insurance pool, or the exposure units, need to be
similar so that a fair premium can be calculated. The fire damage done to brick homes will
ordinarily be less than that of suffered by wooden homes. It would be unfair to combine them
is the same insurance pool and charge each insured the same premium rate based on the
combined losses of the pool. If such as attempt were made, the rate developed would cause
the owners of brick home (less susceptible to loss) to pay too high a premium and the owners
of wooden structures (more susceptible to loss) to pay too low a premium.

2) Accidental and unintentional Loss


This means that if an individual deliberately causes a loss, it should not be paid. If the
intentional loss were paid the effect would be as follows. The loss should be accidental
because the law of large numbers is based on the random occurrence of events. A curious
example of the application of the principle of accidental losses occurs with life insurance, for
which suicide within a year or two of a policy being purchased is considered non accidental.

55
Insurers do not pay such losses. If a suicide occurs several years in after a policy is in force,
however the loss is considered accidental, or the result of mental illness - a cause as
accidental as any other illness.

3) Determinable and Measurable loss


Loss must be definite, measurable and of sufficient severity to cause economic hardship. This
means the loss must be definite to cause time place, and amount. Life insurance in most cases
meets this requirement easily. The cause and time of death can be readily determined in most
cases. It is difficult to determine and measure the losses in some cases. E.g. there are chances
of dishonest claims taking an illness or injury and collecting the insurance payment. It is also
important that the losses insured against be measurable. The company must determine
whether the insured satisfies the definition of disability as stated in the policy, because
sickness and disability are highly subjective. The basic purpose of this requirement is that the
insurers must be able to determine if the loss is covered under the policy, and if it is covered,
how much the company will pay.
4) No Catastrophic Loss / The loss should not the catastrophic
This means that ideally a large proportion of exposure units should not incur losses at the
same time. The pooling technique breaks down if most or all of the exposure units in a certain
class simultaneously incur a loss. Examples of catastrophic losses include, flood, hurricanes,
earth quakes, wild fire, tsunami etc. Insurers ideally wish to avoid all catastrophic losses, but
still employ two approaches to handle the this problem.
1. Re-insurance: that is Insurance companies are indemnified by re-insurers for catastrophic
losses. It is shifting of part or all of the insurance originally written by one insurer to
another.(to be dealt in detail in the seventh chapter)
2. Dispersing coverage over a large geographical area:
This is a technique to reduce the burden of catastrophic losses by dispersing the coverage
area to different geographic locations.

5) Calculable Chance of Loss


The insurer must be able to calculate both the average frequency and the average severity of
future losses with some accuracy. This is necessary so that a proper premium can be charged
that is sufficient to pay all claims and expenses and yield a profit during the policy period.
Certain catastrophic losses, however, are difficult to insure because of the chance of loss
cannot be accurately estimated.

56
6) Economically Feasible Premium
The insured must be able to afford to pay the premium. Premium should be substantially less
than the face value, or amount, of the policy.

 Activity 2

1) Discuss the basic characteristics of insurance


___________________________________________________________________________
___________________________________________________________________________
2) Explain the major requirements of an insurable risk
___________________________________________________________________________
___________________________________________________________________________
___________________________________________________________________________
3.4. Insurance and Gambling

 How could you distinguish Insurance and gambling?

___________________________________________________________________________

! Insurance is often confused with gambling. There are two important differences between

them.

Insurance Gambling

 A technique for handling an already existing  Creates new speculative risk that did not
pure risk. exist before
 Socially productive, since neither the  Socially unproductive, since the winners
insurer nor the insured is placed in a gain comes at the expense of the loser.
position where gain of the winner comes at
the expense
The insurer of the
and the loses have a common interest in the prevention or non occurrence of
insured
loss and the insurer indemnifies the losses incurred by the insured. Whereas, gambling
transaction never restores the losses to earlier financial position. A gambler presumably
enjoys the risk of gambling and therefore would be unlikely to pay the premium needed for
transferring the risk being enjoyed.

57
3.6. Insurance and Speculation

 How could you distinguish Insurance and speculation?

Both are similar in that risk is transferred by a contract and no new risk is created. The main
difference between insurance and speculation lies in the type of that each is designed to
handle, and in the resulting differences in contractual arrangements. The main similarity lies
in the central purpose behind each transaction. However there are some important differences
exist between them.
Insurance Speculation

 Insurance transaction normally involves the  A technique for handling risks those are
transfer of risks that are insurable, since typically not insurable.
the requirements of an insurable risks  Speculation only involves transfer of risks
generally can be met & not reduction of risk. The losses cannot
 Insurance can reduce the objective risk of be predicted based on the law of large
an insurer by application of the law of of numbers.
large numbers

Speculation is a transaction under which one party, for a consideration, agrees to assume
certain risk. The risk of adverse price fluctuation is transferred to speculators who believe
they can make a profit because of superior knowledge of market conditions. The risk is
transferred, not reduced and prediction of loss generally is not based on the law of large
numbers. A speculator is a transferee of risk, and the transferor is usually a business person
wishing to pass on a price risk to someone who is more willing and able to bear it. Such a
business person then is using the transfer method of handling the risk

3.6. Benefits and Costs of Insurance

 Would you estimate the benefits and costs of insurance?

___________________________________________________________________________
3.6.1. Benefits of Insurance

! Insurance has many benefits to different stake holders;

58
1) Uses of insurance to an individual
a. Insurance provides security and safety. Insurance reduces the physical and mental stress
that insured’s face concerning the possibility of death, disability and financial loss. Insured’s,
through transfer of their risk to the insurer reduce their worry about any financial loss they
may face due to accidental misfortune. This means that insured’s are to a large extent certain
that the loss, if at all occurs will be recovered from the insurer.

b. Insurance provides security against loss due to fire in fire insurance. In other types of
insurance, security is provided against loss at a given contingency. Moreover it provides
safety and security against the loss of earning at damage, destructions or disappearance of
property, goods, furniture etc.
c. Insurance affords peace of mind. The knowledge that insurance exists to meet the
financial consequences of certain risks provides a form of peace of mind. This is important
for private individuals when they insure their car, house, possessions and so on, but it is also
of vital importance in industry and commerce. The existence of insurance helps individuals
to have peace of mind and give them relief that eventually makes them stimulated to more
work.

d. Insurance protects mortgaged property. At the death of the owner of the mortgaged
property, or at the time of damage or destruction of the property, the insurer will provide an
adequate amount to the dependents at the early death of the owner to pay off the unpaid
loans, or the mortgage gets a deflated amount at the destruction of the property.

2) Uses of Insurance to Business


a. Reduction of uncertainty
Why should a person put money into a business venture when there are so many risks which
could result in the loss of the money? Yet, if people did not invest in businesses then there
would be fewer jobs, less goods, the need for even higher imports and a general reduction in
wealth. Buying insurance allows the entrepreneur to transfer at least some of the risks of
being in business to an insurer, in the manner we have described earlier. Uncertainty of
business losses is reduced in the world of business. In commerce and industry a huge number
of properties are employed.

b. Increasing business efficiency


Insurance also acts as a stimulus for the activity of businesses which are already in existence.

59
This is done through the release of funds for investment in the productive side of the
business, which would otherwise require to be held in easily accessible reserves to cover any
future loss. Medium sized and larger firms could certainly create reserves for emergencies
such as fires, thefts or serious injuries. However, this money would have to be accessible
reasonably quickly and hence the rate of interest which the company could obtain would be
much less than the normal rate. Quite apart from this is the fact that the money would not be
available for investment in the business itself.

Business efficiency is increased with insurance when the owner of a business is free from
botheration of losses, hence, certainly devote much time to the business. The care free owner
can work better for the maximization of profit. The uncertainty of loss, damage, destruction
or disappearance of a property, may affect the mind of the businessperson adversely. The
insurance, removing the uncertainty, stimulates businesspersons to work hard.

3) Uses of insurance to society


a. Wealth protection
With the advancement of the society, the wealth or the property of the society attracts more
hazards resulting in the creation of new types of insurance invented to protect them against
the possible losses. The present, future and potential property resources are well – protected
through insurance in which each and every member will have financial security against
damage and destruction of wealth. Through prevention of losses, insurance protects the
society against degradation of resources and ensure stabilization and expansion of business
and industry.

b. Economic growth
Insurance provides strong hand and mind and protection against loss of property. In addition
to these, insurance companies accumulate large sum of money available for investment
purpose. Such money accumulated may be invested by the insurance companies themselves
or lent to others to produce more wealth. This will have its contribution to the economic
growth of a country.

The fact that the owner of a business has the funds available to recover from a loss provides
the stimulus to business activity that we noted earlier. It also means that jobs may not be lost
and goods or services can still be sold. The social benefit of this is that people keep their jobs,
their sources of income are maintained and they can continue to contribute to the national

60
economy. We all know the effects on a community when a large employer moves or ceases
operation; the area runs the risk of being depressed, people have less money to spend and the
consequences of this can be far reaching.
To a lesser extent, a major loss resulting in the closure of a business can have the same
impact on a community. It may not be as noticeable as the shut-down of a coal mine or large
factory, but when losses are aggregated throughout the country the effect is considerable. It is
not suggested that insurance alone keeps people in jobs, but it does play a significant role in
ensuring that there are not unnecessary economic hardships.

The three dimensions of benefits that we have already looked at all follow on from the
protection offered by insurance. These benefits may be to the buyer of insurance or to the
economy as a whole, but they relate in some way to the basic idea of providing a risk transfer
mechanism.

3.6.2 Cost of Insurance to Society

! No institution can operate without certain costs. These are listed below so that one can

obtain an impartial view of the insurance institution as a social device. The major social costs
of insurance include the following:
1) Cost of doing business
2) Fraudulent claims
3) Inflated claims

1) Cost of doing the business


The main social cost of insurance lies in the use of scarce of economic resources land, labor
capital and organization to operate the business. In financial terms, an expense loading must
be added to the pure premium to cover the expenses incurred by insurance companies. An
expense loading is the amount needed to pay all expenses, including commissions, general
administrative expenses, state premium taxes, acquisition expenses, and an allowance for
contingencies and profit.

61
! The cost is justified from the insured's view point as follows:

 Uncertainty concerning the payment of a covered loss is reduced because of


insurance.
 The cost of doing business is not necessarily wasteful, because insurers engage in a
wide variety of loss prevention activities.
 The insurance industry provides jobs to millions of workers.
However, because economic resources are used up in providing insurance, a real economic
cost is incurred.
2) Fraudulent claims
These are the claims made against the losses that one caused intentionally by people in order
to collect on their policies. There always exists moral hazard in all forms of insurance. Arson
losses are on the increase. Fraud and vandalisms are the most common motives for arson.
Fraudulent claims are made against thefts of valuable property, such as diamond ring or fur
coat, and ask for reimbursement. These claims results in higher premiums to all insured.
These social costs fall directly on society.
3) Inflated claims
It is a situation where, the tendency of the insured to exaggerate the extent of damages that
result from purely unintentional loss occurrences. Examples of inflated claims include the
following.
a) Attorney for plaintiffs may seek high liability judgments - Liability insurance
b) Physicians may charge above average fees - health insurance
c) Disabled persons may malinger to collect disability income benefits for a longer
duration.
These inflated claims must be recognized as an important social cost of insurance. Premiums
must be increased to cover the losses, and disposable income that could be used for the
consumption of other goods or services is thereby reduced. The social costs of insurance can
be viewed as the sacrifice that society must make to obtain the social benefits of insurance.

 Activity 3

1) Explain how insurance is socially productive while gambling is socially unproductive

62
___________________________________________________________________________
__________________________________________________________________________
2) Explain the benefits of insurance to individual, business, and to society
___________________________________________________________________________
___________________________________________________________________________
___________________________________________________________________________
3) Discuss the cost of insurance to society
___________________________________________________________________________
___________________________________________________________________________

 Check List

Dear learner, below are the most important points which are drawn from the chapter you
have been studying up to now. Put a tick () mark in the box after the point you feel you
have understood. But if you find a concept that you have not mastered yet, please go back
and read the chapter you passed through.

I can:

1) Define insurance and identify the function of insurance

2) Identify the basic characteristics of insurance

3) Discuss the requisites of insurable risks

4) Distinguish Insurance from gambling and speculation

5) Identify the benefits of insurance to individual, business, and society

6) Explain costs of insurance to the society

Chapter Summary
Insurance is one way of risk handling tool which can be applied by organizations and it is the
most formal approach of risk management tool. Insurance is a complicated and intricate
mechanism, and it is consequently difficult to define. However, in its simplest aspect, its
definitions can be considered from four viewpoints that is, from an economic, legal, business,
social, and mathematical. Insurance is an economic system for reducing uncertainty of loss

63
through pooling of losses together, a legal method of transferring risk from the insured to the
insurer in a contract of indemnity, a business undertaking for profit that provides many jobs
in a free enterprise economy, a social device in which the loss of few is covered by the
contribution of many, or an actuarial system of applied mathematics.
Insurance has both primary and secondary functions. Primary functions include; providing
certainty, protection, and risk sharing, and secondary function include prevention of loss and
providing capital. An insurance plan or arrangement typically has certain characteristics.
They include the following: Pooling of losses, payment of fortuitous losses, risk transfer, and
indemnification.
Certain requirements usually must be fulfilled before a pure risk can be privately insured.
From the view point of the insurer, there are ideally six requirements of an insurable risk.
These are, large number of homogeneous exposure units, loss must be accidental and
unintentional, loss must be determinable and measurable, loss should not the catastrophic, the
chance of loss must be calculable, and the premium must be economically feasible.
Insurance has peculiar advantages as a device to handle risk and so ought to be used to bring
about the greatest economic advantages to individuals, business, and society. Even though,
insurance industry provides enormous social and economic benefit, the social costs of
insurance must also be recognized. The major social costs of insurance include operating
expenses, fraudulent claim, and inflated claim.

 Self Test Questions

Part-I: Say “True” for the correct statement and “False” for the incorrect
statement.
1) Insurance is an economic device where by the individual substitutes a small certain cost for a
large uncertain financial loss.
2) Gambling is social productive whereas insurance is socially unproductive.
3) Indemnification means that the insured is restored to his/her approximate financial position
prior to the occurrence of the loss.
4) Insurance has no any cost to the society.
5) A loss that is occurred intentionally by the insured is paid by the insurance company.
6) Sharing of loss is one of the characteristics of insurance that deals with allotment of the
larger financial losses for the entire group.

64
7) If catastrophic loss is occurred the insurer is not obliged to indemnify the insured.
8) Insurance increases fear and anxiety.
9) With the advancement of the society, the wealth or the property of the society attracts more
hazards resulting in the creation of new types of insurance invented to protect them against the
possible losses.
10) Transferring or shifting risk from one individual to a group is the fundamental
characteristics of insurance.

Part-II: Essay Type Questions


1) Explain the basic characteristics of insurance
2) Write the six requisites of insurable risks
3) Explain cost of insurance to the society and which do you think is the most important? Why?

65
CHAPTER FOUR
LEGAL PRINCIPLE OF INSURANCE CONTRACT

Introduction
In the previous chapter, chapter three, we have seen the basics of insurance contract such as:
meaning, function, characteristics, cost and benefits of insurance. Moreover, we tried to
differentiate between insurance, speculation and gambling. Insurance contracts are complex
legal documents that reflect both general rules of law and insurance law. When you buy
insurance, you expect to be paid for a covered loss. Whether you can collect and the amounts
paid are governed by insurance law. Thus, you should have some understanding of the legal
principles and concepts reflected in insurance contracts. Therefore, this chapter will address the
legal part and parcel of insurance contract including the legal principles, legal requirements and
legal unique characteristics of insurance contract. You will understand more clearly the
individual insurance contracts that are discussed later in the consequent chapter if you
understand the legal principles discussed in this chapter.

Objectives
Upon the successful completion of this chapter, student should be able to:

 Describe the legal principles of insurance contract.


 Observe the interdependence that exists between these legal principles
 Appreciate the purposes of each legal principles of insurance
 Realize the legal requirements of insurance contract.
 List and explain the characteristics that makes insurance contract different from others
 Differentiate the application of these legal principles in different insurance contracts
4.1. Principle of Indemnity

 Dear student, would you guess what principle of indemnity states?

___________________________________________________________________________
___________________________________________________________________________
Please now compare your attempt with what real it states.

66
! The principle of indemnity states that a person may not collect more than the actual loss

in the event of damage caused by an insured peril. Thus, while a person may have purchased
coverage in excess of the value of the property, the person cannot make a profit by collecting
more than the actual loss of the property that is destroyed. Many insurance practices result
from this important principle. In general only contracts in property and liability insurance are
subjected to this doctrine, although there are exceptions where statutes have modified its
application.

The principle of indemnity is closely related to insurable interest. The problem in insurable
interest is to determine whether any loss is suffered by a person insured, where as in
indemnity the problem is to obtain a measure of that loss. In the basic fire insurance
contracts, the measure of “actual cash loss” is the current replacement cost of destroyed
property loss plus an allowance for estimated depreciation. The whole purpose is to restore
the insured to his/her former financial position before the happening of the loss. Thus, the
principle eliminates the intention of gambling that incorporates profit motive. Indemnity can
take different forms: cash payment, replacement of the property or reinstatement of the
property.

! The principle of indemnity has two fundamental purposes. The first purpose is to

prevent the insured from profiting from a loss. For example, if Alex's home is insured for
$100,000, and a partial loss of $20,000 occurs, the principle of indemnity would be violated
if $100,000 were paid to him. He would be profiting ($80,000) from insurance.
The second purpose is to reduce moral hazard. If dishonest insured could profit from a loss,
they might deliberately cause losses with the intention of collecting the insurance. If the loss
payment does not exceed the actual amount of the loss, the temptation to be dishonest is
reduced.

! There are several important exceptions to the principle of indemnity. They include the

following:
1) Valued policy: is one that pays the face amount of insurance regardless of actual cash

67
value if a total loss occurs. It is typically used to insure antiques, fine arts, and rare paintings.
2) Replacement cost insurance: means no deduction is taken for depreciation in determining
the amount paid for the loss.
3) Life insurance
A life insurance contract is not a contract of indemnity but is a valued policy that pays a
stated sum of money to the beneficiary upon the insured’s death.

4.2. Principle of Insurable Interest

 Dear student, would you guess what principle of insurable interest mean?

___________________________________________________________________________
___________________________________________________________________________

! A fundamental legal principle underlying all insurance contracts is insurable interest.

Under this principle an insured must demonstrate the existence of financial relationship to
the subject matter insured; otherwise the insured will be unable to collect amounts due when
the insured peril occurs. The principle applies to both life and non-life insurance. The subject
matter insured may include property of value, life of a person, or an event that may cause a
legal liability. For instance: 1) In the case of a property, the owner has a financial interest in
the safety of the property for he/she will suffer a financial loss in the event of destruction of
the property by accidental misfortune. 2) In the case of life insurance, a clear example is the
insurable interest of a wife in the life of a husband and the vice versa. 3) In the business
environment a creditor has a financial interest in the life of a debtor. Thus he has the right to
purchase life insurance policy for the life of the debtor to protect his financial interest.
The doctrine of insurable interest is also necessary to prevent insurance from becoming a
gambling contract.

Insurance follows the person insured and not the property. A policy can be written covering a
certain piece of property and an individual may be named as the one who would suffer a
financial loss if the perils were to occur and cause damage. However, if at the time of the loss
the individual named no longer had an interest in the property, there would be no liability
under the policy. For example suppose that Gemechu owns and insured car, later he sells his

68
car to Obsinet and shortly thereafter the car is destroyed. Gemechu cannot collect under the
policy, because he has no further financial interest in the car since he has sold to Obsinet at a
time of loss he is not an owner of the car. And Obsinet cannot collect because she is not
named as an insured under the policy.

In property and liability insurance it is possible to effect coverage on property in which the
insured doesn’t have an insurable interest at the time the policy is written, but in which such
an interest is expected in the future. In marine insurance a shipper often obtains coverage on
cargo it has not yet purchased in the anticipation of buying cargo for a return trip. As a result
the courts generally hold that in property insurance, insurable interest need exist only at the
time of the loss and not at the time in inception of the policy.
In contrast, in life insurance, the insurable interest requirement must be met only at the time
of inception of the policy, not at the time of death. Life insurance is not a contract of
indemnity but is a valued policy that pays a stated sum upon the insured’s death. Since the
beneficiary has only a legal claim to receive the policy proceeds, the beneficiary does not
have to show that a loss has been incurred by the insured’s death. For example, if Meti takes
out a policy on her husband’s life and later gets a divorce, she is entitled to the policy
proceeds upon the death of her former husband if she has kept the insurance in force. If she
continues to maintain the insurance by paying the premiums, she may collect on the
subsequent death of her former husband even though she is remarried and suffers no
particular financial loss upon his death. It is sufficient that she had an insurable interest when
the policy on an employee who is no longer with the firm. A creditor may retain the policy on
the life of a debtor who has repaid his/her obligation. The insurable interest requirement must
be met only at the inception of the contract.

Purposes of an insurable interest

! To be legally enforceable, all insurance contracts must be supported by an insurable

interest. Insurance contracts must be supported by an insurable interest for the following
reasons.
 To prevent gambling
 To reduce moral hazard
 To measure the amount of the insured's loss in property insurance

69
First, an insurable interest is necessary to prevent gambling. If an insurable interest were
not required, the contract would be a gambling contract and would be against the public
interest. For example, one could insure the property of another and hope for a loss to occur.
One person could similarly insure the life of another person and hope for an early death.
These contracts clearly would be gambling contracts and would be against the public interest.

Second, an insurable interest reduces moral hazard. If an insurable interest were not
required, a dishonest person could purchase a property insurance contract on someone else's
property and then deliberately cause a loss to receive the proceeds. But if the insured stands
to lose financially, nothing is gained by causing the loss. Thus, moral hazard is reduced.

Finally, in property insurance, an insurable interest measures the amount of the insured's
loss. Most property contracts are contracts of indemnity, and one measure of recovery is the
insurable interest of the insured. If the loss payment cannot exceed the amount of one's
insurable interest, the principle of indemnity is supported.

 Activity 1

1) Explain the principle of indemnity.


___________________________________________________________________________
___________________________________________________________________________
2) What is an insurable interest? Why is an insurable interest required in every insurance
contract?
___________________________________________________________________________
4.3. Principle of Subrogation

 How do you think principle of subrogation is applicable in insurance contract?

___________________________________________________________________________

! The principle of subrogation strongly supports the principle of indemnity. Subrogation

means substitution of the insurer in place of the insured for the purpose of claiming
indemnity from a third person for a loss covered by insurance. The insurer is entitled to

70
recover from a negligent third party and loss payments made to the insured. For example, a
negligent motorist fails to stop at a red light and smashes into Ato Alemu's car, causing
damage in the amount of 5000 Br. If he has collision insurance on his car from Ethiopian
Insurance Corporation, his company will pay the physical damage loss to the car and then
attempt to collect from the negligent motorist who caused the accident, the insured gives to
the insurer legal rights to collect damages from the negligent third party.

Purposes of Subrogation
Subrogation has three basic purposes. First, subrogation prevents the insured from collecting
twice for the same loss. In the absence of subrogation, the insured could collect from the
insurer and from the person who caused the loss. The principle of indemnity would be
violated because the insured would be profiting from a loss.
Second, subrogation is used to hold the guilty person responsible for the loss. By exercising
its subrogation rights, the insurer can collect from the negligent person who caused the loss.

Finally, subrogation helps to hold down insurance rates. Subrogation recoveries can be
reflected in the rate making process, which tends to hold rates below here they would be in
the absence of subrogation.
Importance of subrogation

! Dear student, you should keep in mind five important corollaries of the principle of

subrogation.
1) The general rule is that by exercising its subrogation rights, the insurer is entitled only to
the amount it has paid under the policy.
2) The insured cannot impair the insurer’s subrogation right. The insured cannot do anything
after a loss that prejudices the insurer’s right to proceed against a negligent third party.
3) The insurer can waive its subrogation rights in the contract. To meet the special needs of
some insured’s, the insurer may waive its subrogation rights by a contractual provision for
losses that have not yet occurred.
4) Subrogation does not apply to life insurance and to most individual health insurance
contracts.
5) The insurer cannot subrogate against its own insured’s.

71
4.4. Principle of Utmost Good Faith

 Dear student, how do you think principle of utmost good faith is applicable in

insurance contract?
___________________________________________________________________________

! Insurance contract is based on the principle of utmost good faith that is, a higher degree

of honesty is imposed on both parties to an insurance contract than is imposed on parties to


other contracts. Thus, the principle of utmost good faith imposed a high degree of honesty on
the applicant for insurance.
The principle of utmost good faith in insurance is said to be a contract of utmost good faith.
In effect, this principle imposes a higher standard of honesty on parties of insurance
agreement than is imposed on ordinary commercial contracts. Insurance contracts are based
on mutual trust. This means that both the insured and the insurer must make full disclosure
of material facts that have a bearing on the assessment of the risk. Intentional concealment,
misrepresentations and fraud may lead to the avoidance of the insurance contract. The insured
is bound to give all the facts having material effect on the assessment of risk. The application
of this principle can be expressed in representation, concealment, warranties, and mistake.

Representations: are statements made by an applicant for insurance before the policy is
issued. Although the representation need not be in writing, it is usually embodied in a written
application. An example of a representation in life insurance would be answering yes or no to
a question as to whether or not the applicant had been treated for any physical condition or
illness by a doctor within the previous five years. If you apply for life insurance, you may be
asked questions concerning your age, weight, height, occupation, state of health, family
history, and other relevant questions. Your answers to these questions are called
representations.
Concealment: is intentional failure of the applicant for insurance to reveal a material fact to
the insurer. It is the same thing as nondisclosure; that is, the applicant for insurance
deliberately withholds material information from the insurer.
Warranties: are statements of fact or a promise made by the insured, which is part of the
insurance contract and which must be true if the insurer is to be liable under the contract. It is

72
in an insurance contract holding that before the insurer is liable, a certain fact, condition or
circumstance affecting the risk must exist.
Mistakes: when an honest mistake is made in a written contract of insurance, steps cab be
taken to correct it after the policy is issued. Generally, a policy can be reformed if there is
proof of a mutual mistake or a mistake on one side that is known to be a mistake by the other
party, where no mention was made of it at the time the agreement was made. A mistake in
the sense used here does not mean an error in judgment by one party but refers to a situation
where it can be shown that the actual agreement made was not the one stated in the contract.

4.5. The principle of contribution

 Dear learner, would you guess if one property insured by more than one insurance

company, which insurance company should pay for insured during loss?
___________________________________________________________________________

! This also supports the principle of indemnity. It is applied to a situation where a person

or firm, for some reasons, purchase insurance from two or more insurers to cover the same
subject matter/property against loss or damage. Under such circumstance, the insured cannot
collect compensation from each insurer. If this happen, insurance becomes a profit making
mechanism. So, the insured is paid only to the extent of the loss he has suffered. But, each
insurer will make contribution to settle the claim. The contribution may be proportional
amount based on the sum insured under the respective insurers. However, to know if an
insured has more than one insurer for the same risk, especially in countries like ours could
be difficult.

! Some important principle of contribution in property and liability insurance include:

1) Pro Rata liability clause: it is a generic term for a provision that applies two or more
policies of the same type cover the same insurable interest in the property. Each insurer’s
share of the loss is based on the proportion that its insurance bears to the total amount of
insurance on the property. The pro rata clause typically states that if more than one policy is
in force on a given piece of property, each policy will pay in the ratio of the face value of

73
each policy divided by the total amount of insurance in force on the property. For example,
assume that Tolera owns a building and wishes to insure it for Birr 200,000. For under
writing reasons, insurers may limit the amount of insurance they will write on a given
property. Assume that an agent places Birr 100,000 of insurance with Ethiopian Insurance
Corporation, Birr 50,000 with Awash Insurance Company, and Birr 50,000 with Oromia
Insurance Company. If a Birr 10,000 loss occurs, each company will pay only its pro rata
share of the loss as described in table below.
Table 4.1. Pro rata liability clause
Insurer Pro rata Liability Clause Compensation
Ethiopian Insurance Corporation Birr 100,000 X Birr 10,000 Birr 5000
Birr 200,000
Awash Insurance Company Birr 50,000 X Birr 10,000 Birr 2,500
Birr 200,000
Oromia Insurance Company Birr 50,000 X 10,000 Birr 2,500
Birr 200,000
Total paid Birr 10,000

Therefore, Tolera would collect Birr 10,000 for the loss and not Birr 30,000.

2) Contribution by Equal Shares: with this method, each insurer shares equally in the loss
until the share paid by each insurer equals the lowest limit of liability under any policy, or
until the full amount of the loss is paid. For example, assume that the amount of insurance
provided by companies Ethiopian Insurance Corporation, Awash Insurance Company, and
Oromia Insurance Company is Birr 100,000, Birr 200,000, and Birr 300,000 respectively. If
the loss is birr 150,000 each insurer pays an equal share of Birr 50,000.
Table 4.2.a. Contribution by Equal Shares
Insurance Companies Amount of insurance Contribution by equal Total paid
shares
Ethiopian Insurance Birr 100,000 Birr 50,000 Birr 50,000
Corporation
Awash Insurance Birr 200,000 Birr 50,000 Birr 50,000
Company
Oromia Insurance Birr 300,000 Birr 50,000 Birr 50,000
Company

74
However, if the loss were Birr 500,000, how much would each insurer pay? In this case,
each insurer would pay equal amounts until its policy limits are exhausted. The remaining
insurers then continue to share equally in the remaining amount of the loss until each insurer
has paid its policy limit in full, or until the full amount of the loss is paid. Thus, the share of
each insurer is shown in the following table.

Table 4.2.b. Contribution by Equal Shares


Insurance Companies Amount of insurance Contribution by equal Total paid
shares
Ethiopian Insurance Birr 100,000 Birr 1000,000 Birr 100,000
Corporation
Awash Insurance Birr 200,000 Birr 100,000+100,000 Birr 200,000
Company
Oromia Insurance Birr 300,000 Birr 100,000+100,000 Birr 200,000
Company

4.6. The Principle of Doctrine of proximate cause

 Would you explain the principle of doctrine of proximate cause?

___________________________________________________________________________

The maxim ‘ Causa proxima non remota spectature’ means that proximate (nearest) cause
and not the remote one is to be taken notice of at the time of determining the liability of the
insurer. The insurer is not liable for remote cause even if it is one of the insured risks for the
occurrence of the risk of which the insured is to be compensated. The insurer is liable to
make the payment of loss under the policy, otherwise not. The insured may recover the loss
from the insurer only when:
 The loss has been caused by the insured peril; and
 The cause has been proximate to the loss.
Therefore, the insurer is not liable for the loss due to a proximate, which is not an insured
peril.

75
4.7. Valid Elements or Requirement of an Insurance Contract

 What are the requirements that insurance contract must fulfill?

___________________________________________________________________________
_____________________________________________________________________

! An insurance policy is based in the law of contracts. To be legally enforceable, an

insurance contract must meet four basic requirements: offer and acceptance, consideration,
competent parties, and legal purpose.
1) Offer and Acceptance
The first requirement of a binding insurance contract is that there must be an offer and an
acceptance of its terms. In most cases, the applicant for insurance makes the offer, and the
company accepts or rejects the offer. An agent merely solicits or invites the prospective
insured to make an offer.
2) Consideration
The second requirement of a valid insurance contract is consideration the value that each
party gives to the other. The insured's consideration is payment of the first premium (or a
promise to pay the first premium) plus an agreement to abide by the conditions specified in
the policy. The insurer's consideration is the promise to do certain things as specified in the
contract. This promise can include paying for a loss from an insured peril, providing certain
services, such as loss prevention and safety services, or defending the insured in a liability
lawsuit.
3) Competent Parties
The third requirement of a valid insurance contract is that each party must be legally
competent. This means the parties must have legal capacity to enter into a binding contract.
Most adults are legally competent to enter into insurance contracts, but there are some
exceptions. Insane persons, intoxicated persons, and corporations that act outside the scope of
their authority cannot enter into enforceable insurance contracts. Minors normally are not
legally competent to enter into binding insurance contracts; but most states have enacted laws
that permit minors to enter into a valid life insurance contract.
The insurer must also be legally competent. Insurers generally must be licensed to sell

76
insurance in the state, and the insurance sold must be within the scope of its charter or
certificate of incorporation.
4) Legal Purpose
A final requirement is that the contract must be for a legal purpose. An insurance contract
that encourages or promotes something illegal or immoral is contrary to the public interest
and cannot be enforced. For example, a street pusher of heroin and other illegal drugs cannot
purchase a property insurance policy that would cover seizure of the drugs by the police. This
type of contract obviously is not enforceable because it would promote illegal activities that
are contrary to the public interest.

4.7.1. Unique characteristics of insurance contracts


Insurance contracts have distinct legal characteristics that make them different from other
legal contracts. Several distinctive legal characteristics have already been discussed. As we
noted earlier, most property and liability insurance contracts are contracts of indemnity; all
insurance contracts must be supported by an insurable interest; and insurance contracts are
based on utmost good faith.

! Other distinct legal characteristics are as follows:

a. Aleatory contract
b. Unilateral contract
c. Conditional contract
d. Personal contract
e. Contract of adhesion
a. Aleatory contract
An insurance contract is Aleatory rather than commutative. An Aleatory contract is a contract
where the values exchanged may not be equal but depend on an uncertain event. Depending
on chance, one party may receive a value out of proportion to the value that is given. For
example, assume that Mr “z” pays a premium of $500 for $100,000 of home owners
insurance on his home. If the home were totally destroyed by fire shortly thereafter, he would
collect an amount that greatly exceeds the premium paid. On the other hand, a homeowner
may faith fully pay premiums for many years and never have a loss.
In contrast, other commercial contracts are commutative. A commutative contract is one in
which the values exchanged by both parties are theoretically equal. For example, the

77
purchaser of real estate normally pays a price that is viewed to be equal to the value of the
property.
b. Unilateral contract
An insurance contract is a unilateral contract. A unilateral contract means that only one party
makes a legally enforceable promise. In this case, only the insurer makes a legally
enforceable promise to pay a claim or provide other services to the insured. After the first
premium is paid, and the insurance is in force, the insured cannot be legally forced to pay the
premiums or to comply with the policy provisions. Although the insured must continue to pay
the premiums to receive payment for a loss, he or she cannot be legally forced to do so.
However, if the premiums are paid, the insurer must accept them and must continue to
provide the protection promised under the contract.
In contrast, most commercial contracts are bilateral in nature. Each party makes a legally
enforceable promise to the other party. If one party fails to perform, the other party can insist
on performance or can sue for damages because of the breach of contract.
c. Conditional contract
An insurance contract is a conditional contract. That is, the insurer's obligation to pay a claim
depends on whether the insured or the beneficiary has complied with all policy conditions.
Conditions are provisions inserted in the policy that qualify or place limitations on the
insurer's promise to perform. The conditions section imposes certain duties on the insured if
he or she wishes to collect for a loss. Although the insured is not compelled to abide by the
policy conditions, he or she must do so to collect for an insured loss. The insurer is not
obligated to pay a claim if the policy conditions are not met. For example, under a
homeowner’s policy, the insured must give immediate notice of a loss. If the insured delays
for an unreasonable period in reporting the loss, the insurer can refuse to pay the claim on the
grounds that a policy condition has been violated.
d. Personal contract
In property insurance, insurance is a personal contract, which means the contract is between
the insured and the insurer. Strictly speaking, a property insurance contract does not insure
property, but insures the owner of property against loss. The owner of the insured property is
indemnified if the property is damaged or destroyed. Because the contract is personal, the
applicant for insurance must be acceptable to the insurer and must meet certain underwriting
standards regarding character, morals, and credit.
A property insurance contract normally cannot be assigned to another party without the
insurer's consent. If property is sold to another person, the new owner may not be acceptable

78
to the insurer. In contrast, a life insurance policy can be freely assigned to anyone without the
insurer's consent because the assignment does not usually alter the risk or increase the
probability of death.
e. Contract of adhesion
A contract of adhesion means the insured must accept the entire contract, with all of its terms
and conditions. The insurer drafts and prints the policy, and the insured generally must accept
the entire document and cannot insist that certain provisions be added or deleted or the
contract rewritten to suit the insured. Although the contract can be altered by the addition of
endorsements or other forms, the endorsements and forms are drafted by the insurer. To
redress the imbalance that exists in such a situation, the courts have ruled that any
ambiguities or uncertainties in the contract are construed against the insurer. If the policy is
ambiguous, the insured gets the benefit of the doubt.

 Activity 2

1) Explain the principle of subrogation. Why is subrogation used?


___________________________________________________________________________
___________________________________________________________________________
2) Explain the principle under which utmost good faith is used.
___________________________________________________________________________
__________________________________________________________________________

3) Assume wezero Ayantu has the following four liability insurance policies: Birr 300,000
with Global Insurance Company, Birr 300,000 with Oromia Insurance Company, Birr
700,000 with United Insurance Company, and Birr 900,000 with Nice Insurance Company.
How much will each policy pay for a Birr 2,000,000 loss?
a) If they use prorate liability clause
b) If they use contribution by equal shares
___________________________________________________________________________
___________________________________________________________________________
3) What are the unique characteristics of insurance contracts?
___________________________________________________________________________
___________________________________________________________________________

79
 Check list

Dear learner, below are the most important points which are drawn from the chapter you
have been studying up to now. Put a tick () mark in the box after the point you feel you
have understood. But if you find a concept that you have not mastered yet, please go back
and read the chapter you passed through.

I can:

1) Explain insurance contract and its legal principles

2) Define principle of indemnity and its purpose

3) Explain principle of insurable interest and its application

4) Differentiate principle of subrogation and utmost good faith

5) Identify valid requirement of insurance contract and its unique features

Chapter Summary
The four chapter of this module enabled you to identify the legal principles of insurance
contract, requirements of insurance contracts, unique features of insurance contract. The legal
principles of insurance contract include principle of indemnity, insurable interest,
subrogation, utmost good faith, contribution, and proximate cause.
The principle of indemnity states that a person may not collect more than the actual loss in
the event of damage caused by an insured peril. It has two fundamental purposes to prevent
the insured from profiting from a loss and to reduce moral hazard. Valued policy,
replacement cost insurance, and life insurance are exception to the principle of indemnity.

Under the principle of insurable interest an insured must demonstrate the existence of
financial relationship to the subject matter insured; otherwise the insured will be unable to
collect amounts due when the insured peril occurs. To be legally enforceable, all insurance
contracts must be supported by an insurable interest. Insurance contracts must be supported
by an insurable interest for the following reasons; to prevent gambling, to reduce moral
hazard, to measure the amount of the insured's loss in property insurance.

80
Principle of subrogation means substitution of the insurer in place of the insured for the
purpose of claiming indemnity from a third person for a loss covered by insurance. It has the
purpose of prevents the insured from collecting twice for the same loss, to hold the guilty
person responsible for the loss, and helps to hold down insurance rates.

Principle of utmost good faith is that a higher degree of honesty is imposed on both parties to
an insurance contract than is imposed on parties to other contracts. The application of this
principle can be expressed in representation, concealment, warranties, and mistake.
Principle of contribution is applied to a situation where a person or firm, for some reasons,
purchase insurance from two or more insurers to cover the same subject matter/property
against loss or damage. During such situation insurer either pays by pro rata liability clause or
contribution by equal shares.
An insurance policy is based in the law of contracts. To be legally enforceable, an insurance
contract must meet four basic requirements: offer and acceptance, consideration, competent
parties, and legal purpose.

 Self Test Questions

Part-I: Say “TRUE” for the correct statement and “FALSE” for incorrect statement
1) Principle of indemnity is applicable for life insurance.
2) Subrogation means substitution of the insurer in place of the insured for the purpose of
claiming indemnity from a third person for a loss covered by insurance.
3) The insurer can subrogate against its own insured’s.
4) There is a possibility that one property insured in more than two insurance companies.
5) Insurance is commutative type of contracts.
Part-II: Choose the best answer among the given alternatives
1) ____________ is statements made by an applicant for insurance before the policy is
issued.
A. Representations C. Warranties
B. Concealment D. Mistake
2) Which of the following is/are not the unique feature of insurance contract?
A. It is aleatory contract D. It is personal contract
B. It is bilateral contract E. None
C. It is conditional contract

81
3) The actual value of building in Adama Town owned by Mr. X was 1millionETB had a fire
insurance coverage from say Ethiopian Insurance Corporation (EIC). Unfortunately, the
building caught fire and 25% of the building was burnt out which was estimated by the EIC
inspection agent. The compensation and claim settlement officer had offered the face value of
the building. Based on the information given, which principle of insurance actually violated?
A) Principle of Subrogation D) Principle of Utmost Good Faith
B) Principle of Indemnity E) None
C) Principle of Contribution
4) Assume that a 60,000 birr liability claim is covered under two liability insurance contracts.
Policy ‘A’ has 100,000 birr limit of liability for the claim, while policy ‘B’ has 25,000 birr
limit of liability. If both contracts provide for contribution by equal shares how much will
each insurer contribute toward the above claim?
A) Both are 30,000 birr
B) ‘A’ pays 25,000 birr and ‘B’ pays 35,000 birr
C) ‘A’ pays 35,000 birr and ‘B’ pays 25,000 birr
D) ‘A’ pays 12,000 birr and ‘B’ pays 48,000 birr
5) “Insurance contract is the one that makes only one party legally enforceable to its promise,
while the rest party has already fulfilled its obligation”. This characteristic of insurance
contract is?
A. Aleatory
B. Unilateral
C. conditional
D. Personal

82
CHAPTER FIVE
LIFE AND HEALTH INSURANCE
Introduction
So far in this module we have seen the nature of risk and how to manage them by applying
different mechanisms so as reduce and/or eliminate the burden on our business venture or
individual’s equity. Furthermore, we have also tried to see insurance in some details about the
definition, principles, legal requirements, unique characteristics and basic parts of insurance
contractual agreement. Now in this chapter we will take a look, specifically, at life and health
insurance contracts, which we have considered as more important to be discussed in this
course.
Human values, aside from being more important to us from a personal stand point, are more
important than all the different property values combined. Therefore, in this chapter we will
discuss two broad sections related with human values. In the first section you will learn about
life insurance policy details such as, under writing life insurance, premature death, ways of
providing life insurance, major types of life insurance, premium determination in life
insurance. The second section deals with health insurance specifically medical expense and
disability income contracts.

Objectives
At the end of this unit, you should be able to:

 Understood premature death, and economic justification of buying life insurance


 The different ways of acquiring life insurance as a customer or prospect insured
 Describe the major types of life insurance and their uses
 Appreciate the advantages and disadvantages of major life insurance types.
 Explain health insurance and distinguish it from life insurance
5.1. Life Insurance

 Dear learner, could you explain what life Insurance is?

___________________________________________________________________________
__________________________________________________________________________

83
! Life insurance has many definitions among others let us see few of them:

Definition 1
Life insurance is a contract whereby the insurer for certain sum of money or premium
proportionate to the age, profession, health and other circumstances of the person whose life
is insured engage that if such person dies with in the period specified in the policy the insurer
will pay the amount specified by the policy according to the term there of to the person in
whose favor the policy was entered to.
Definition 2
Life insurance is a social and economic devise by which a group of persons may cooperate to
ameliorate/make better the loss resulting from the premature death of members of the group.
The insuring organization collect contributions from each member, invest this contribution,
grants both their safety and a minimum interest return and distribute benefits to the estates of
the members who die.

The main purpose of life insurance is financial protection to the dependants of the insured
upon the premature death of the insured. The sum assured is, then, upon the death of the
insured will be paid to the beneficiaries. The financial compensation will provide security for
a certain period of time. The insured may also purchase life insurance policy with such
objectives as settling personal loans and other debts. If the insured dies before settling his/her
debts, the insurer will settle the debt outstanding to the creditors, hence protecting the family
from financial loss.

Life insurers are generally engaged in the provision of both protection and saving. The
protection is against financial loss difficulty and is acquired for a consideration called
premium, which is the price that keeps the policy in force. The protection given by the
insurer is death benefits to the beneficiary of the insured, or in the case of survival of the
insured, other financial benefits in accordance with the policy contract.

Meaning of Premature Death


Premature death is defined as the death of family head with outstanding unfulfilled financial
obligation, such as dependants to support, children to educate and a mortgage to pay off.
Premature death can cause serious financial problems for the surviving family members
because their share of diseased breadwinner’s earnings is lost forever. If the replacement

84
income after the death of the family head, from different sources is or if the accumulated
financial assets available to the family are inadequate the surviving family members are
expected to suffer great financial insecurity.

! There are some costs that are associated with premature death. These costs are:

1) The family’s share of the earning is lost forever


2) Additional expenses may also be incurred because of funeral expenses, uninsured medical
bill.
3) Because of insufficient income, some families will experience a reduction in their standard
of living
4) Certain non economic costs are incurred, such as the diseased bread winner’s counseling
and guidance for his/her children, loss of parental role model and emotional grief.

! Therefore, life insurance is used to alleviate the financial consequence of premature

death. The purchase of life insurance is economically justified if the insured earns an income,
and others are dependent on that earning capacity for at least part of their financial support. If
the family head dies prematurely, life insurance can be used to restore the family’s share of
the diseased family head’s earnings.

It should be noted that a life insurance, unlike property and liability insurance, is a valued
policy that pays a stated sum to a named beneficiary and is not a contract of indemnity (see
the exceptions of the principle of indemnity in chapter 4). The insurer event is the uncertainty
of the time of death. As a social and economic device life insurance is a method by which a
group of people may cooperate to share the loss resulting from the premature death of
member of the group. The insurer collect contributions from each member, invests these
contributions, guarantees both their safety and a minimum interest return, and distributes
benefits to the estates of the members who die.
5.1.1 Unique Features life insurance

! Life insurance is a risk-pooling plan, an economic device through which the risk of

premature death is transferred from the individual to the group. However, the contingency

85
insured against has characteristics that make it unique; as a result, the contract insuring
against this contingency is different in certain respects from other types of insurance.
 The benefits are determined in advance. The insured decides for him/herself the
amount of insurance protection he needs. The insurer will then decide on the
corresponding reasonableness of the amount of coverage and sets the corresponding
premium.
 The amount of money required to pay the death benefits in a given period are to be
collected in advance so that there should not be shortage of funds to pay claims as
they occur.
 Each insured in the group should be charged an appropriate premium, which reflects
the amount of risk he/she brings to the group. In other words, losses are to be
distributed among the group of insured’s in an equitable manner.
 The probability of claim increases with the passage of time since insured’s exhibit
deteriorating health condition as they grow old.
 In addition to protection against uncertainty, life insurance has the function of
accumulation of money/saving.
 Life insurance is not strictly a contract of indemnity for the value of a person cannot
be precisely put in financial terms.
 There is no possibility of partial loss in life insurance as there is the case of property
and liability insurance. Therefore, all policies are cash payment policies. In the event
that a loss occurs, the company will pay the face amount of the policy.
 The provision of life assurance is a quite different process from the provision of non-
life insurance. The main distinction is that in life assurance the event being ensured is
either certain to happen, in the case of those policies paying on death, or scientifically
calculable, in the case of policies not paying a benefit on death.
In addition to these there are a number of special features, which are worth mentioning
at this stage:
a) Premium payments: Life assurance premiums are payable by level amounts throughout
the period of the policy. This means that each person pays the same amount throughout, that
amount being determined by his/her age on affecting the policy. Premiums can be paid
annually, semi-annually, quarterly or monthly and are often met by standing orders with
banks whereby the policyholder instructs his/her bank to make the appropriate payments at

86
the correct times. It is also possible for the insured to pay premiums for a specified period of
time or even a single payment at lump sum at the time the policy is purchased.
b) Participation in profits: Life assurance companies value their assets and liabilities at
regular intervals, say every year or others every three years. This valuation of their operation
allows them to determine whether any surplus exists after calculating all future liabilities and
allowing for other contingencies. Should such a surplus exist, it is distributed among those
policyholders who have 'with-profits' or 'participating' policies. Such policies allow the
policy holder to participate in any profits the company makes. It does not guarantee a bonus
to each policyholder, as the company may not have a surplus, but it does mean that any
available surplus will be distributed.

The policyholder pays an additional amount for the privilege of participating in profits. The
bonuses are then added to the sum assured and payable at the maturity date. They can be
either simple reversionary bonus, that are computed at a rate percent on the basic sum
assured, or compound reversionary bonuses, that are computed at a rate percent of the basic
sum assured plus any existing bonus payments already declared.
c) Surrender values: When a person no longer wants his/her policy, or for some reason
cannot continue the premiums, he/she can ask for the surrender value
d) Investments: We have already identified the life assurance industry as being of
considerable size by considering the number of policies in force and the value of premiums
paid each year. These vast amounts of money are held by companies to meet future liabilities
and are termed life assurance funds. These funds do not lie dormant waiting for claims to
come in; rather they are invested to provide income for the companies and so assist
policyholders and shareholders. Not only do these two groups benefit, but the country as a
whole benefits.
5.1.2. Types of Life Insurance Policies

 Would you distinguish types of life insurance policies?

_________________________________________________________________________
Life insurance policy can be classified as either term insurance or cash value life insurance.
Term insurance provides temporary protection, while cash-value life insurance has a saving
component and builds cash value at the end. Numerous combinations of these two types of
life insurance are also available today.

87
! This sub-section will have a discussion on the following major classes of life insurance:

1. Term life insurance.


2. Whole life insurance.
3. Endowment life insurance and.

1. Term Life Insurance


When a life insurer sells a term life insurance policy, it promises to pay the beneficiary if the
insured dies within a specified period. If the insured outlines (survives) to the end of the
period, the insured makes no payment

Term life insurance is similar to property insurance in this respect. If there is no loss to home
or automobile while the policy is in force, the insurer makes no payment. This is also the case
with term life insurance. Moreover, like property insurance policies, term life insurance does
not build saving or cash value, as do other types of life insurance. Thus, term life insurance is
often spoken of as providing “pure death protection”. This type of insurance is relatively
simple type of insurance, and in part of this reason, it has been among the first insurance
which is provided through internet.
Characteristics of Term Insurance
 The period of protection is temporary
 It is renewable & convertible
 Have no cash value or saving element
The period of protection provided under term life insurance is only for temporary period,
such as one, two, five, ten or twenty years, unless the policy is renewed, the protection
expires at the end of the period.

Most term life insurances are renewable, which means that the policy can be renewed for
additional periods without evidence of insurability. Here the premium is increased at each
renewal and is base on the insured attained age. The purpose of renewal provision is to
protect the insurability of the insured and to protect the policy from lapsing.

Most term insurance policies are also convertible, which means that the policy can be
exchanged for a cash value policy without evince of insurability. There are two methods for
converting a term insurance these are “attained age” and “original age method”. Under the

88
attained-age method, the premium charged is based on the insured attained age at the time of
protection. Under the original-age method, the premium charged is based on the insurers
original age when the term insurance was first purchased.
Finally, term insurance policies have no cash value or saving element, although some long-
term policies develops a small resent it is used up by the contract expiration date.
Types of Term Insurance
Insurance sell several types of term life insurance policies including the following major
categories:
 Yearly renewable
 5,10,15,20-year term to age 65
 Decreasing value term insurance
 Increasing value term insurance
 Reentry term insurance
Yearly renewable term insurance is issued for a one-year period, and the policy owner can
renew for a successive one-year periods to same stated age without evidence of insurability.
Premiums increase with the increase in age but level with in the given protection period (one
year).
Term life insurance can also be issued for e, 5, 10, 15, or 20 years or for a longer period.
Like yearly renewable term insurance the premiums paid during the term period are level, but
it increase when the policy is renewed. A term to age 65 policy provides protection to age 65,
and then it expires.
A decreasing value term life insurance policy provides the beneficiary with less and lesser
proceeds each year while the policy is in force. That is, if the death occurs in the first policy
years, the beneficiary receives the face amount. If the death occurs in a succeeding years the
proceeds will be less. Here the premiums paid during the term period are level but purchase
less insurance. That is, the mount of death benefits decreases because the chance of death
increases with age. Decreasing term insurance is useful to provide funds to repay a mortgage
or provide support for a dependent child until the child reaches age, 21. Over time, the fund
needed to accomplish each of these goals decreases and so the insurance proceeds.
Hypothetical life insurance company illustration of decreasing value term insurance for
Mr. Z

89
Table: 5.1. Illustration for a 5 year decreasing value term insurance policies
Age/sex: 35, male , smoker
Year Age Annual premium Death benefit
1 35 $100 $90,000
2 36 $100 $80,000
3 37 $100 $90,000
4 38 $100 $60,000
5 39 $100 $50,000
An increasing term insurance policy provides a proceed that increases each year. If death
occurs in the first year, the insurer pays the face amount of the policy, and if the insured dies
in the succeeding years the proceeds will increase accordingly. Increasing term insurance
policy is attractive in an inflationary economy. For example, as the price of a collage
education increases so do the insurance procedures of increasing term insurance. Premium for
these policy increases at each renewal.
Hypothetical life insurance company illustration of increasing value term insurance for
Mr. X. Table 5.2 Illustration for a 5 year increasing value term insurance policies
Age/sex: 35, male , smoker
Year Age Annual premium Death benefit
1 35 $120 $100,000
2 36 $120 $110,000
3 37 $120 $120,000
4 38 $120 $130,000
5 39 $120 $140,000
A level term insurance policy pays the same amount of benefit if the death occurs at any
point while the policy is enforced. Table 5.3 below presents a sale illustration for a 5 year
level term policy prepared for Mr. Alemu in a hypothetical life insurance company. As can be
seen the premium and the death benefits are level throughout the period of protection. Even at
the end of 5th year the policy may be kept (renewed) but the premiums will then rise steeply
to reflect Mr. Alemu age at that time.

Hypothetical life insurance company illustration of level term insurance for Mr. Alemu
Age/sex: 35, male, non-smoker.

90
Table: 5.3. Illustration for a 5 year level term insurance policies
Age/sex: 35, male , smoker
Year Age Annual premium Death benefit
1 35 $114 $100,000
2 36 $114 $100,000
3 37 $114 $100,000
4 38 $114 $100,000
5 39 $114 $100,000

Reentry term is another important term insurance product. Under this policy selection and
protection of the prospect insured is based on evidence (lower) 0f morality rate, if the
insured can periodically demonstrate acceptable evidence of insurability and good health,
protection will be kept if not the insured will be protected no more years.
Advantages of Purchasing Term Life Insurance
 When the need for protection is temporary
 For low income earners. If the amount of income that can be spent on life insurance is
limited. Term insurance is appropriate and cost effective to be used.
 Finally term insurance can also be used to insure guarantee for future insurability.
Disadvantages of Term Life Insurance
 Term insurance premiums increases with age and eventually it reaches at a level
which is not affordable.
 No saving element or cash value at the end of the policy. Term insurance is not
appropriate if you wish to save money for a specific need.

2. Whole Life Insurance


In contrast to term insurance, which provides only temporary protection without any cash
value, whole life insurance provides both life time protection and cash value or saving
element at the age of 100. Whole life insurance is considered as matured when the insurer
makes a claim payment. In addition to the claim payment the insurer knows for a certainty
that eventually pay a claim on every whole life policy which remains in force. This
circumstance makes whole life insurance different from term life insurance contracts.

Here you may ask a question that where this cash value or saving element comes from the
following discussion will give you the answer. The level premium method of paying whole

91
life insurance produces a saving element called cash value, in permanent life insurance
policies such as whole life insurance; the saving element is generated as follows. The insurer
initially charges a substantially larger premium than necessary to be paid for financing early
death claims. This additional charge continues during the first portion of the policies
duration. After a period this additional change and the compound interest on them generates a
significant saving value.
Types of Whole Life Insurance
Based on the method of premium payment insurers classify whole life insurance policies in
the following categories
i) Single premium whole life insurance
ii) Continuous premium whole life insurance
iii) Limited payment whole life insurance

i) Single premium whole life insurance


Single premium whole life insurance policies are those to which the insurer promises to pay
the claim whenever death occurs in exchange to one relatively large premium. Individuals
did not usually prefer this type because it is difficult to provide a single large amount of
premium.
ii) Continuous –Premium Whole Life Insurance
This policy requires insured’s to pay the same premium as long as they live or until they
reach age 100. Insurance companies usually sales this policy as; level premium whole life
insurance, straight premium whole life insurance, or as ordinary life insurance. The
premiums take in to account, mathematically, both compound interest and the mortality rate
(probability of the insured’s death) if the insured survives up to 100 age the insurer will give
the cash value. In other word, the insured death means the insurer must make a claim
payment. It also means an insured will no longer make premium payment to the insurer.

iii) Limited-Payment Whole Life Insurance


It is another class of whole life insurance where the premium payment schedule falls
somewhere between single premium and continuous premium policies. The insurance is
permanent, and the insured has life time protection. The premiums are also level but they are
paid only for a certain period. For instance, a man age 35 may purchases 20 year limited
payment insurance for a specified amount say $25,000. After 20 years, the policy is
completely paid up, and no additional premiums are required even though the coverage

92
remains in force.
Paid up policy should not be confused with a matured policy. When we say a policy is
matured, it means the face amount is paid as a death claim or as an endowment. But when we
say the policy is paid-up it is to mean that, payment of premium is exhausted but not the
policy is ended rather the policy is in force up to age 100.

The size of each payment is a function of the number of time it will be paid. The fewer the
number of payment, the larger each payment will be. Thus, a man age 35, payment for a 20
year limited payment whole life insurance becomes paid up within 20 years. Customers
waiting long term death protection but who don’t want to continue paying life insurance
premiums, especially during retirement, choose limited-payment policy. Here the secret of
the limited-payment policy is that the insurer is willing to accept a smaller number of larger
payments, as a result, greater compound interest is earned compared to the continuous
premium plan.
3. Endowment Insurance
Endowment insurance is another type of life insurance which combines the characteristics of
both term and cash-value policy. An endowment policy pays the face amount of insurance if
the insured dies within a specified period. If the insured survives to the end of the
endowment period, the face, amount is paid to the policy owner at that time. For instance, a
man of age 35 who purchases 20 year endowment policy and died any time with in the 20
year period the face amount would be paid to his/her beneficiary; if he/she survives to the end
of the period, the face amount (the cash value or saving element) would be paid to him.

Like term insurance, endowment insurance contract provide death benefit for a specified
period of time. However unlike term insurance, endowment insurance has a cash value and
the policy owner is paid the contracts face amount at the end of term of protection if the
insured is still alive.

5.1.3. Factors Considered in Life Insurance Underwriting

 Dear student, would you guess factors to be considered in life insurance underwriting?

__________________________________________________________________________

93
! Quite often, life insurance covers are aimed at protecting the dependents of the insured

from suffering financial losses in case of his premature death. This means that the covers are
usually connected with the occurrence of death to the insured. As a result in life insurance
underwriting the factors that are considered or examined are those that influence morality the
insured himself/herself. These include age, sex, current physical condition, personal medical
history, family medical history, occupation, habits, avocation, marital status, and the like.

1. Age: This is the most important factor to consider in life insurance underwriting. The
likelihood of death or illness generally increases with an increase in age. People develop
physical problems as their age increases. The higher the age level, the greater they develop
physical problems as their age increases.
2. Gender: Empirical studies indicate that women generally live longer than men. This may
influence the underwriter to charge a lower premium payments for women for a given age
level as their counterparts, men.
3. Current Physical Condition: This refers to the proposed insured’s current physical and
health condition regarding pulse rate, heart condition, blood pressure, lungs, nervous
system, body build, height weight, etc…
4. Personal Medical History: The insured’s past medical history is examined to check for
any previous illness that may possibly reoccur in the future. From personal health history
of the insured the underwriter wants to know whether the proposed insured had gone
major medical treatments in the past.
5. Family Medical History: Here, the medical history for the insured family is examined to
discover any possible hereditary diseased or deficiencies.
6. Occupation: Occupation is given as much equal importance as that of age in life
insurance underwriting. Occupation can affect the insured’s chance of suffering accidents
of premature death. For example a coal miner is much more exposed to risk of premature
death or illness than a manager or an accountant.
7. Insurable Interest: There should be an insurable interest to be protected by purchasing
life insurance policy. Absence of insurable interest could reflect the presence of wagering
in the contract. The underwriter must make sure that there is an insurable interest in
applying for life insurance cover. This involves identifying any relationship between the
proposed insured and named beneficiary.

94
8. Financial Position: The proposed insured’s financial position or his level of income has
become an important factor to consider in life insurance underwriting. The proposed
insured’s financial position is examined to check the existence of speculation and lack of
insurable interest. Particularly, close examination is to be made when the amount of cover
is appreciable large.
9. Habits: Habits such as drug or alcohol consumption and smoking could lead to accidents
by retarding a person’s judgment, reducing flexibility and damaging his reflex system.

Sources of Information for Underwriting

! The assessment and evaluation of the risk is based on the information collected by the

underwriter. Pertinent information needed for underwriting is obtained from the following
sources:
 Proposal Form  Questionnaires and Interview
 Medical Report  Underwriting Manuals
 Attending Physicians Statement  Inspection Report
 Agent’s/ Salesman’s Report

 Activity 1

1) Define life insurance and list the unique characteristics of life insurance.
___________________________________________________________________________
___________________________________________________________________________
2) Explain types of life insurance
___________________________________________________________________________
___________________________________________________________________________

3) What are the factors to be concerned in life insurance underwriting?

___________________________________________________________________________
___________________________________________________________________________
________________________________________________________________________

95
5.1.4. Premium Determination in Life Insurance

 How would you determine premium in life insurance?

__________________________________________________________________________

Dear student, before any extended discussion on life insurance premium calculation, it seems
quite logical for you to know about the types of premiums available in life insurance. To
this effect, we have mentioned two types of life insurance premiums.

1) Net premium: The determination of net premium considers only the mortality rate and
rate of interest. It ignores operating costs charged by the insurer. N.B. Net premium provides
the insurer only with the amount of money required to pay death claims. The net premium to
be paid could be single or level premium. Net single premium is the net premium to be paid
as a single sum at the beginning of the contract while a net level premium is a premium
charge that doesn't change from year to year throughout the term of the policy.
2) Gross premium: The insurer's costs of operating the business are added to the net
premium, which is called loading. Loading is the act of adding costs of running business to
the net premium costs including operating expenses, commissions, advertisement expenses,
etc.

! Life insurance rates are influenced by three major determinants:

1) Expected mortality rates in the insured population


2) Investment income earned by the insurer on invested premium income, and
3) Expenses incurred in operating an insurance enterprise and in providing insurance-related
services.
a. Computing Net Single Premium
The information required to compute net single premium are the rate of interest and amount
of insurance policy. For the purpose of illustration, the following assumptions are made.
 Population size =10,000
 Male people at age 40 = 9,580
 Probability of death at age 40 = 165/9580= 0.017
 Amount of insurance policy (Benefit) = 5000

96
Assume further that premium will be collected at the beginning of the policy issue. Death
benefit (policy amount) is to be paid at the end of the year.

 Interest rate is 10%.


 Each policyholder brings the same level of risk to the group
 Expected number of death at age 40 is 165.
 Expected amount of death benefits would be = 165X5000= 825,000
N.B. The insurer will not collect 825,000 from the insured but only its present
value.
 Present value of Birr 825,000 = 825,000/ (1.10) = Birr 750,000 is the total net
premium to be collected.
 Net single premium = Total net premium/Number of insured
= 750,000 /9,580
= 78.288
The insurer collects 750,000 Birr and invests it at 10%. An amount of 750,000 birr collected
at the current period will just be enough to settle the expected death claims (825,000 birr) by
the end of the year.

Premium computation for term life insurance


Consider the following information
 3 years term policy
 Policy amount = 5,000 birr to be paid at the end of the year.
 Number of policyholders at age 30 = 958,000.
 Interest rate = 10%
 Premium = Single premium payment at the beginning of the year.

 Mortality rate table, male, 1990.

Year Age Number Number Probability of


of living of dying Dying
1 30 958,000 1657 0.00173
2 31 956,343 1702 0.00178
3 32 954,641 1747 0.00183

97
Expected Death Claims

Number Policy Expected death


Year Age of Dying Amount claims
1 30 1657 5,000 8, 285,000
2 31 1702 5,000 8,510,000
3 32 1747 5,000 8,735,000

Total expected death claims 25,530,000

Discount each year's expected death claim at 10% interest rate to arrive at the present value of
total clams, (see below).

1 2 3 4 5 6
(2*3) (4/5)
Year Death Claims PV Factor at PV of Claims No. of Annual net
10% Insured Premium

1 8,285,000 0.9091 7,531,893.5 958,000 7.862


2 8,510,000 0.8264 7,032,664 958,000 7.341
3 8,735,000 0.7513 6,562,605.5 958,000 6.850

25,530,000 21,127,163 22.053

Net single premium = PV of Claims


Number of insured’s
= 21127163/958,000
= 22.053 birr
Each insured will be required to pay a net single premium of Birr 22.053 at the beginning of
the policy. This premium enables the insurer to meet the expected death claims that occur in
each year. See the amortization table below.

98
b. Net Level Premium
Instead of a single premium payment, in this case the policyholders will pay annual
premiums of equal size. Consider the following three points carefully.

i. Not all the policyholders will pay the annual level premiums (Since some of them are
expected to die before the end of the term).
ii. The insurer will collect a limited amount of money to invest at the very beginning of
the policy.
iii. The total annual level premiums paid by a policyholder under the level scheme will be
greater than the single net premium.

Assuming all facts as in the previous illustration, calculate the net level premium?
Example
Year Age No. of
Insured
1 30 958,000
2 31 956,343
3 32 954,640
Assume each insured pays level premium of Birr 1 throughout the term of the policy.

Year Age No. of Present value of Birr.1 Present value of


Insured payable at beginning of Birr.1 premium
year

1 30 958,000 1 958,000
2 31 956,343 0.9091 869,411
3 32 954,641 0.8264 788,915
Total 2,616,326
Payment per insured = 2,616,326/958,000
= 2.731
Net level premium = Net Single Premium
PV. Of Birr 1 premium
payment per insured
= 22.05/2.731
= Birr 8.075

99
The amount of net level premium to be collected and the expected death claims are shown
below.

Year Annual Total Beginning Beg. Bal Death Ending


level premium balance invested at 10 claims balance
premium collected %
1 8.075 7,735,850 7,735,850 8,509,435 8,285,000 224,435
2 8.075 7,722,470 7,946,905 8,741,596 8,510,000 231,596
3 8.075 7,708,726 7,940,322 8,734,354 8,735,000 (646)*
* Difference due to rounding.
Present value of Birr 1 payment = (958,000/958,000) 1+ (956,343/958,000) 0.9091 +
(954,640/958,000)0.8264
= 2.731
Gross Premium
Gross premium is the premium charged for insurance that includes anticipated cost of losses,
overhead, and profit. The net level premium for life insurance represents the pure premium
that is unadjusted for the expenses of doing business. The pure premium is actually the
contribution that each insured makes to the aggregate insurance fund each year for the
payment of both death and living benefits. The gross premium is determined by adding a
loading allowance to the net level premium. The loading must cover all operating expenses,
provide a margin for contingencies, and provide for a contribution to profits.

 Activity 2

1) You are given certain data (simplified below)


Age Number of Number of Present value of Birr 1 at 2%
living dying
Year Factor
25 1000 10 1 0.98
26 990 12 2 0.96
27 978 13 3 0.94
a) Calculate the net single premium for a two year term insurance policy of Birr 1000 issued
at age 25 b) Calculate the net level premium for the same policy.

100
8.2. Health Insurance

 Could you distinguish health insurance from life insurance?

___________________________________________________________________________
Dear learner, in the first section of this chapter we have seen life insurance and its economic
justification, types of life insurance, how to determine premium in life insurance and other
details, now in this section we will deal the details of health insurance.
The sections specifically deals with policies providing reimbursement for medical expenses
and policies replacing lost income that people cannot earn when they are disabled by
accident or illness traditionally called disability insurance

! A serious illness or injury can result in great financial and economic insecurity; two

major problems are encountered if the sickness or disability is prolonged and severe. The
disabled person loses his or her work earnings, and medical expanse must also be paid. Dear
learners, disability resulting from illness or accident may be even greater peril to a family
than premature death. This is because disability not only cuts off income but also may create
large medical expenses. A six month or longer period of disability in fact, is a more likely
cause of loss to people on their working years that are premature death. Insurance can transfer
the burden of the costs of illness or accident so that insured don’t face financial mainly
because of poor health.
Health insurance provides a wide variety of specific individual health insurance coverage’s.

! Among the varieties the following are selected to be death in this section

5.2.1. Medical expense insurance


5.2.2. Disability income insurance
5.2.3. Workers compensation insurance
5.2.1. Medical Expense Insurance

 Dear learner, could you guess what medical insurance means?

___________________________________________________________________________

101
! As we have explained above medical expense provides for the payment of the cost of

medical care that results from sickness and injury. Its benefits help to meet the expenses of
physical hospital nursing, surgical expense, and related services, as well as medications and
supplies. The benefits may be in the form of reimbursement of actual expanses up to
specified limit of insurance, cash payment or the direct provision of services. Medical
expenses insurance is paid under the following specific coverage’s

1. Hospital insurance 3. Physicals expense insurance


2. Surgical insurance 4. Major medical insurance.

1. Hospital Insurance
Hospital insurance contract is one of the basic health insurance policies. Hospital insurance
pays for medical expenses incurred while the insured is in a hospital. A typical hospital
insurance policy provides two basic benefits.

 Daily hospital benefit


 A benefit for miscellaneous expense

A daily benefit is paid for room and board changes while hospitalized the plan typically pay
for a stated number of days. For example, the insured may be allowed to select a daily
benefit of Br.175 or Br. 200 for 90,120 or 360 days.

! There are three basic approaches for paying the daily room and board benefit.

 Indemnity approach: The plan pays the actual costs of the daily services up to some
maximum limit.
 Valued approach: A fixed amount is paid for each day of hospitalization regardless
of the actual cost of the services provided.
 Service approach: Service benefits rather than cash benefits are provided to the
insured. For example, the full cost of hospital services in a semiprivate room may be
paid for each day of hospitalization up to some maximum number of days.

102
A hospital policy also provides a lump-sum benefit for miscellaneous expenses, such as
laboratory charges, X- ray drugs, and the use of the operating room. Depending on the plan,
part or all of the miscellaneous expenses are paid up to some maximum limit. There are
several methods for determining the amount paid for miscellaneous expenses.

Choice of maximum dollar amounts: Many plans offer the insured a choice of maximum
dollar amounts, such as Br.2000 or Br.3000.
Multiple of daily room benefit: Another approach is to pay a benefit for miscellaneous a
expense that is a multiple of the daily room benefit, such as 10, 15, or 20 times the daily
benefit.
Percentage participation clause: Another method is to use a percentage participation clause
(coinsurance). Such as payment of 80 percent of the miscellaneous expenses up to some
maximum limit.
With respect to maternity benefits, individual hospital policies generally cover only the
complications of pregnancy, and not each normal costs of childbirth. A complication of
pregnancy is a medical condition that is distinct form a normal pregnancy, but which is
caused by the pregnancy.

2. Surgical Expense Insurance


Individual hospital-surgical plans also insurer coverage for surgical expenses, such as
physicians’ fees associated with covered surgeries. Surgical expense insurance can be added
to a hospital policy. There are three basic approaches for compensating physicians under
surgical expense policy.
i. Surgical schedule: The benefits paid can be determined by a schedule of surgical
operations. A surgical schedule lists the different surgical procedures and the maximum
amount paid for each operation. The insured has a choice of surgical schedules, such as a
Br.1200 or Br.1500 schedule. For example, the policy may pay Br.2000 for a gallbladder
operation but only Br.200 for a tonsillectomy.

ii. Relative value schedule: A relative value schedule can also be used to determine the
maximum amounts paid for surgical operations. Instead of a stated dollar amount for each
surgical procedure, a relative value schedule assigns a number of units or points for each
operation based on the complexity of the operation. The actual dollar amount paid for each
operation is determined by multiplying the number of units by the value of the unit (stated in
the policy). The advantage of this approach is that it determines the amounts paid based on

103
the complexity and degree of difficulty of each operation. For example, a complex eye
operation may have a value of 100, while a simple tonsil operation may have a value of 15
thus, if the unit value is Br.10, the policy pays Br.1000 for the eye operation but only Br.150
for the tonsil operation.
Another advantage of a relative value schedule is that it can be conveniently adapted to
differences in the cost of living and in surgeons’ fees in different geographical areas.
iii. Reasonable and customary charges: Another method for compensating physicians is on
the basis of their reasonable and customary charges. Reasonable and customary charges (also
referred to as usual, reasonable, and customary charges) are charges that fall within a range of
fees charged by physicians for a similar medical procedure in the same geographical area. If
the actual charge falls within the range of allowable fees, insurers usually pay the charge in
full, subject to any deductible or coinsurance requirements. There is no uniform method for
determining reasonable and customary charges and insurers differ in the methodology used.

Surgical expense plan can be reimbursed on the basis of schedule approach, reasonable and
customary. Under Schedule charges approach; common surgical procedures are listed in a
schedule with a maximum dollar amount paid for each procedure. However, schedule
procedure with listed maximum fees becomes obsolete quickly because of the increase in
surgical fees over time,
Under reasonable and customary charges approach, surgeons are reimbursed based on their
normal fees as long as the fee is reasonable and customary.

3. Physician’s Expense Insurance


Physician’s expense insurance pays a benefit for non-surgical care provided by a physician in
the hospital, the patient’s home, or in the doctor’s office. Some plans also pay for diagnostic
X-ray and laboratory expenses performed outside the hospital. It also pays for visits to a
doctor’s office or for a doctor house calls or hospital visits, usually with a limit per visit.
Example Br 25 to Birr 100 and a maximum number of calls per sickness or injury.

4) Major Medical Insurance


Insured’s often desire broader and more comprehensive protection than that is provided by
the basic medical expense coverage’s just discussed. An alternative approach of obtaining the
desired protection is the purchase of major medical policy. This insurance is also designed to
pay a large proportion of the covered expenses of a catastrophic illness or injury.

104
5.2.2. Disability-Income Insurance

 Dear learner, could you guess what Disability income insurance means?

___________________________________________________________________________

Disability-income insurance is another important form of individual health insurance. A


serious disability can result in a substantial loss of works earnings. Unless you have
replacement income from disability-income insurance or income from other sources, you may
be financially insecure. Many workers seldom think about the financial consequences of a
long-term disability. However, the probability of becoming disabled before age 65 is much
higher than is commonly believed, especially at the younger ages.

! The financial loss to the family from long-term total and permanent disability can be

substantially greater than the financial loss that results from premature death at the same age.
In the case of premature death, the family loses its share of the deceased family head's future
earnings, and funeral expenses are also incurred. However, in the case of long-term total and
permanent disability, earned income is lost; medical bills are being incurred; savings are
reduced or depleted; employee benefits may be lost; and additional expenses are incurred,
such as getting someone to care for the disabled person. It is clear that disability-income
insurance should be a high priority in a personal risk management program.

Generally this insurance provides income payments when the insured is unable to work
because of sickness or injury. An individual policy pays monthly income benefits to an
insured who becomes totally disabled from a sickness or accident. The amount of disability
insurance you can buy is related to your earnings. To prevent over insurance and to reduce
moral hazard and malingering, most insurers limit the amount of disability income sold to no
more than 60 to 80 percent of your gross earnings.

Meaning of Total Disability


The most important policy provision in a disability income policy is the meaning of "total
disability.” Most policies require the worker to be totally disabled to receive benefits.

105
Definitions of Total Disability

! There are several definitions of total disability. The most important include the following

 Inability to perform all duties of the insured’s own occupation


 Inability to perform the duties of any occupations for which the insured is reasonably
fitted by education, training, and experience
 Inability to perform the duties of any gainful occupation loss of income test

The, first, most liberal definition defines total disability in terms of occupation. Total
disability is the complete inability of the insured to perform each and every duty of his / her
occupation. An example would be a surgeon whose hands blown off in a hunting accident.
The surgeon could no longer perform surgery and would be totally disabled under this
definition.

The second definition is more restrictive intricate, total disability is the complete inability to
perform the duties of any occupation for which the insured is reasonably fitted by education,
training, and experience. Thus, if the surgeon who lost a hand in a hunting accident could get
a job as a professor in a medical school or as a research scientist, he or she would not be
considered disabled because these occupations are consistent with the surgeon's training and
experience.
The third definition is the most restrictive and is commonly used for hazardous occupations
where disability is likely to occur. Total disability is depended as the inability to perform the
duties a gainful occupation. The courts generally have interprets this definition to mean that
the person is totally disabled if he or she cannot work in any gainful occupation reasonably
fitted by education, training, and experience.
Finally, some insurers use a loss-of-income test to determine if the insured is disabled. You
are considered disabled if your income is reduced as a result of a sickness or accident. A
disability-income policy containing this definition typically pays a percentage of the
maximum monthly benefit equal to the percentage of earned income that is lost. For
example, assume that Firaoli earns birr 1000 monthly and has disability-income contract with
a maximum monthly’ benefit of Birr 800. If Firaoli work earnings are reduced to birr 500
monthly because of disability (50persent) the policy pays birr 400 monthly (50persent of birr
800).

106
5.2.3. Workers’ Compensation Insurance
Workers’ compensation insurance covers the loss of income and the medical and
rehabilitation expenses that result from work-related accidents and occupational disease. The
standard workers’ compensation and employers’ liability policy is really two policies in one
workers’ compensation insurance and employers’ liability insurance.

 Activity 3

1) Define medical insurance and under what specific coverage it is paid?


___________________________________________________________________________
___________________________________________________________________________

2) Compare and contrast the three basic approaches for compensating physicians under
surgical expense policy.
___________________________________________________________________________
__________________________________________________________________________

3) Define disability income insurance with practical example


___________________________________________________________________________
___________________________________________________________________________

 Check list

Dear learner, below are the most important points which are drawn from the chapter you
have been studying up to now. Put a tick () mark in the box after the point you feel you
have understood. But if you find a concept that you have not mastered yet, please go back
and read the chapter you passed through.

I can:

1) Define life insurance and explain its economic purposes

2) Explain premature death death and identify costs related to it

3) Identify unique characteristics of life insurance

4) Explain type of life insurance

107
5) Identify factors considered in life insurance underwriting

6) Determine premium for life insurance

7) Define health insurance

8) Distinguish health insurance for life insurance

Chapter Summary
Human value is much more important than the total value of the property owned. The value
may be considered in terms of financial and non financial aspects. As a matter of fact
individuals may have a regular or daily income keeping their life alive and their health good.
Such individually may still have some other dependant family member who is dependant in
full or partially on their income.
Life insurance is a social and economic devise by which a group of persons may cooperate to
ameliorate/make better the loss resulting from the premature death of members of the group.
Life insurance has many unique features that make it differ from non life insurance. The need
for having life insurance for persons having the above persons and family characteristics is
mandatory this is because in the absence of life and health insurance dependants may suffer a
lot financial anxiety because of damage, sickness or passing away of the bread winner. It is
not also mean that individuals having no dependant wouldn’t need to have life and health
insurance in fact, they needs to have but the extent of the necessity of having the coverage
varies with the personal status to status, income to income , family to family of the bread
winner. There are three major types of life insurance. These are term life insurance, whole
life insurance, and endowment life insurance. In underwriting of life insurance factors such as
age, gender, current physical condition, personal medical history, family medical history,
occupation, and others are considered.
Life insurance rates are influenced by three major determinants: Expected mortality rates in
the insured population, investment income earned by the insurer on invested premium
income, and expenses incurred in operating an insurance enterprise and in providing
insurance-related services. Life insurance premium is calculated by either net premium or
gross premium method.
Health insurance provides a wide variety of specific individual health insurance coverage’s.
Among the varieties of the medical expense insurance, disability income insurance, and
workers compensation insurance are few.

108
 Self Test Questions

Part-I: Say “TRUE” for the correct statement and “FALSE” for incorrect statement

1) In endowment insurance, the sum insured is always payable only up on the death of
insured.

2) Life insurance is a contract of indemnity.

3) Hospital insurance pays for medical expenses incurred while the insured is in a hospital.

4) Net premium is the premium charged for life insurance that includes anticipated cost of
losses, overhead, and profit.

5) Single premium whole life insurance is a type of whole life insurance mainly preferred by
many insured.

6) Surgical expense plan can be reimbursed on the basis of schedule, reasonable and
customary approaches.
7) Workers’ compensation insurance covers the loss of income and the medical and
rehabilitation expenses that result from work-related accidents and occupational disease
8) Term insurance is appropriate if you wish to save money for a specific need.

9) The main purpose of life insurance is financial protection to the dependants of the insured
upon the premature death of the insured.

10) Age is the most important factor to consider in life insurance underwriting.

Part-II: Choose the best answer among the following alternatives

1) Which of the following factor(s) can not affect premium in life insurance underwriting?

A. Age D. Personal medical history

B. sex E. None of the above

C. Current physical condition

2) If Ato Gemechu, age 35 purchased a 20 years life insurance and died any time within the
20 years period the face amount would be paid to his/her beneficiary but if he survives to the
end of the period nothing is given to him. Then, the type of life insurance purchased by Ato

109
Gemechu is_______________
A. Term D. Annuity
B. Whole E. None
C. Endowment
3) Which of the following is not correct about term life insurance?
A. As it is a temporary protection there is savings
B. It does not provide loans nor do cash surrender value
C. The premium payment is to be made only once
D. Its cost is relatively low
E. None
4) Under hospital insurance, the basic approach in which daily room pays the actual costs of
the daily services up to some maximum limit is _______________
A) Valued approach C) Service approach
B) Indemnity approach D) None
5) ________refers to the process of selecting and classifying applicants for insurance.
A. Production C. Underwriting
B. Re-insurance D. Rate making E. None

Part-III: Short Answer Questions


1) What are the sources of information for underwriting?
2) Under what specific coverage medical insurance are paid?
3) List the major types of life insurance. Which one do you prefer if you want to purchase life
insurance? Why or why not?

CHAPTER SIX

110
NON-LIFE INSURANCE
Introduction
Dear learner, as insurance has developed and grow, the various types of coverage have been
grouped into several classes, which have come about by practice within insurance company
offices, and by the influence of legislation controlling the financial aspects of transacting
insurance. Insurance offices are generally split up into departments or sections, each of which
will deal with types of risk, which have an affiliation with each other. There is a very wide
variety in the way in which companies organize their business, but the following are the
commonly type of non life insurance that will be discussed in this chapter. These are motor
insurance, burglary and housekeeping insurance, fire and lighting insurance, marine
insurance, aviation insurance, liability insurance, pecuniary insurance, fidelity guarantee
insurance, and engineering insurance.

Objectives
After studying this chapter, you should be able to:
 Differentiate commercial auto policy and private auto policy
 Analyze the events covered under the standard fire policy
 Distinguish fidelity and surety bonds insurance
 Explain ocean marine insurance and distinguish it from aviation insurance
 Distinguish fidelity and surety bonds insurance
 Explain the function of liability insurance
 Explain engineering insurance
6.1. Motor Insurance/Automobile Insurance

 Would you explain what motor insurance means?

___________________________________________________________________________
Most automobile insurance contracts are schedule contracts that permit the insured to
purchase both property and liability insurance under one policy.

! The minimum requirement by law is to provide insurance in respect of legal liability

to pay damages arising out of injury caused to any person by automobile. Policies with

111
various levels of cover are available:

 Third party only: provides cover in respect of liability incurred through death or
injury to a third party, or damage to third party property.

 Third party, fire and theft: provides cover as above and in addition includes cover for
damage to the vehicle from fire or theft.

 Comprehensive: provides cover as above and in addition including cover for


accidental loss of, or damage to, the vehicle itself. This is the most common form of
automobile insurance policy.

Private car insurance applies to private cars used for social and domestic purposes and/or
business purposes. Comprehensive policies issued to individuals also include personal
accident benefits for the insured and spouse, medical expenses and loss of, or damage to,
rugs, clothing and personal effects.

Vehicles used for commercial purposes (including Lorries, taxis, vans, hire cars, milk floats
and police cars) are not insured under private car policies, but under special contracts known
as commercial vehicle policies.

Separate cover is available for motorcycles. The type of policy depends upon the machine,
whether it is a moped or a high-powered motorcycle, and on the age and experience of the
cyclist. The cover is comparatively inexpensive relative to motorcar insurance.

Special policies are offered to garages and other people within the motor trade, to ensure that
their liability is covered while using vehicles on the road. Damage to vehicles in garages and
showrooms can also be included under such policies.

In addition to private cars, motorcycles and commercial vehicles, there are a number of
vehicles which fall into a category known to insurers as 'special types'. These will include
forklift trucks, mobile cranes, bulldozers and excavators. Such vehicles may travel on roads
as well as building sites and other private ground. Where these vehicles are not used on roads
and are transported from site to site, it is more appropriate to insure the liability under a
public liability policy, since the vehicle is really being used as a 'tool of trade' rather than a
motor vehicle and include fire, theft, collision and a wide range of other perils.

! Two standard automobile insurance contracts cab be used by businesses. The first is

112
commercial vehicle policy and the second is private vehicle policy.
a. Commercial vehicle policy/ Business auto Policy
This policy covers losses to vehicles. The losses that are not covered by this policy are:
 Losses arising outside the stated geographical area.
 Wear and tear, depreciation of the vehicle, Mechanical or electrical break down.
 Loss due to over-loading or strain
 Loss due to racing, pace making and speed testing.
 Loss caused due to driving under the influence of intoxicating liquor or drug.
 Loss due to explosion.
 Loss due to damage on any bridge or road caused by load of vehicle.
 Loss due to natural perils, war, invasion, mutiny strikes martial law, riots etc.
b. Private Vehicle Policy/ Personal Auto Policy
This policy is the same as the commercial vehicle policy but only with additional perquisites.
This policy in addition to the commercial vehicle polices doesn't cover:

 Loss occurring in connection with bodily injury if the number of persons exceeds the
sitting capacity.
 Loss directly occasioned by aircrafts or other aerial factors while traveling at
sonic/supersonic speeds.
6.2. Burglary, Robbery, and Theft Insurance

 Do you guess what Burglary and housekeeping insurance means?

___________________________________________________________________________

! As used in insurance contracts, the meaning of the terms burglary, robbery, and theft are

important in understanding the extent of coverage. These terms always refer to crimes by
persons other than the insured, officers or directors of the insured, or employees of the
insured, coverage on which provided by fidelity bonds.

Burglary is defined to mean unlawful taking of property from within premises closed for
business, entry to which has been obtained by force.
Robbery is defined to mean unlawful taking of property from another person by force, by

113
threat of force, or by violence. Personal contact is the key to understanding the basic
characteristic of the robbery peril.
Theft is a broad term that includes all crimes of stealing, robbery, or burglary. It is a catch all
term and is not distinguished from breaking and entering. Thus, any stealing crime not
meeting the definition of burglary or robbery is theft.

! Dear student, according to the Ethiopian Insurance Corporation (EIC) burglary policy it

does not cover losses or theft committed by:


1) Members of the insured’s household
2) The insured him/herself or his assignee
3) Theft connected with war (declared or undeclared or any kind of population uprising, or
4) Theft of values including documents and works of art unless agreed per hand. In addition,
failure to disclose material facts at the time of writing the policy will also make any theft
claims null and avoid

 Activity 1

1) Discuss the loss not covered under commercial vehicle insurance and private vehicle
insurance?
___________________________________________________________________________

2) Distinguish Burglary, Robbery, and theft insurance?


___________________________________________________________________________
___________________________________________________________________________
6.3. Fire and Lighting Insurance

 Dear learner, would you define fire insurance?

___________________________________________________________________________

114
! This insurance covers losses caused by fire and lightning. The losses are only property

losses, this insurance doesn't cover items that can be covered by other insurance policies like
life insurance for life. This policy doesn't cover the following aspects. Loss or damage to
property occasioned by its own fermentation, natural heating, spontaneous combustion, or by
its undergoing any heating or drilling process.

Loss or damage in consequence of the burning of property by order of any public authority or
subterranean fire. Loss caused directly or indirectly by nuclear weapons’ material ionizing
radiations, contamination by radio activity or waste from combustion of nuclear fuel. Loss
caused by natural perils, invasion, war, mutiny, riots, strikes, martial law etc.

 Loss occasioned by the burning of forests, bushes jungle, etc.


 Loss occurring due to goods held in trust.
In most commercial policies the insured will require cover for buildings, machinery and
plant, and stock. These are the three main headings under which property is insured and in
some cases a list of such items can run to many pages, depending upon the size of the insured
company.
6.4. Marine Insurance

 Dear learner, would you define marine insurance?

___________________________________________________________________________

Modern commerce has caused insurance to develop and attain the high degree of refinement
it has today. As world trade grew and values at risk became larger, the need for coverage
became more apparent. Larger ships and more advanced instruments of navigation made long
voyages possible, and with these changes came the realization that insurance protection was
almost a necessity.

! Ocean marine insurance provides protection for goods transported over water. Ocean

marine policies relate to three areas of risk: the hull, cargo and freight.
Hull insurance covers physical damage to the ship or vessel. It is insurance for the ship.

115
Cargo insurance covers the shipper of the goods if the goods are damaged or lost. It is
insurance for property being transported.
Freight insurance indemnifies the ship owner for the loss of earnings (transportation fee) if
the goods are damaged or lost and are not delivered. When goods are lost by marine perils
then freight or part of it is lost; hence the need for coverage of transportation fee that the ship
owner can gain is compensated by the insurer.

The risks against which these items are normally insured are collectively termed 'perils of
the sea’. Cargo is usually insured on a warehouse (of departure) to warehouse (of arrival)
basis and frequently covering all risks. Terms of sale and conditions of carriage have
important implications for cargo insurers where goods may change ownership and pass
through the hands of more than one shipper. It is vitally important in cargo insurance to
establish who is responsible for the insurance coverage and to work out when the risk passes
from the consignor/seller to the consignee/purchaser.

Marine liabilities insurance


The custom has been to provide insurance for three-quarters of the ship owner’s liability for
collisions at sea under a marine policy. The remaining quarters, and all other forms of
liability, are catered for by associations set up for the purpose by ship owners and known as
protecting and indemnity clubs (P and I clubs). It should be noted that the P and I clubs can
now insure hull and machinery as well as liabilities.

6.5. Aviation Insurance

 Dear learner, would you define aviation insurance?

___________________________________________________________________________
The use of aircraft as a means of transport is increasing each year and because of the
specialist and technical nature of the risks associated with it, plus the high potential cost of
accidents, all aviation risks, from component parts to complete jumbo jets are insured in the
aviation insurance market. Most policies are issued on an 'all risks' basis, subject to certain
restrictions. The buyers of these policies include the large commercial airlines, corporate
aircraft owners, private owners and flying clubs.

! Like automobile insurance, aviation insurance includes both property insurance on the

116
planes and liability insurance. Aviation insurance policy provides physical damage coverage
for damage to air craft, liability coverage for injury to passengers and people on the ground,
and medical expenses coverage for passengers.

6.6. Liability Insurance

 Dear learner, would you define liability insurance and what it covers?

___________________________________________________________________________

! Liability insurance contracts obligate the insurer to pay amounts the insured becomes

legally obligated to pay as a result of covered injury to others. In addition, the insurer is
obligated to provide legal defense services for the insured against liability claims to which the
coverage applies.
Basis of Legal Liability
Each person has certain legal rights. A legal wrong is a violation of a person’s legal rights, or
a failure to perform a legal duty owed to a certain person or to society as a whole.
The peril of legal liability arises out of the general rule of law that people are responsible for
any loss (injury) they cause another to suffer. The law creates three categories for describing
situations in which one person injures another:
 Torts, or civil wrongs
 Breaches of contracts (This can occur when one party fails to keep a legally binding
agreement.)
 Criminal acts. (These actions are forbidden by law and can result in fines,
imprisonment, or other penalties).

 Activity 2

1) Define fire insurance and discuss perils covered under fire insurance
___________________________________________________________________________
___________________________________________________________________________
2) Explain ocean marine and aviation insurance
___________________________________________________________________________

117
___________________________________________________________________________
___________________________________________________________________________
6.7. Pecuniary Insurance
Pecuniary insurance is coverage for monetary loss due to a wide range of factors from
external and internal influences. Specifically, under this coverage only money lost is payable
upon proof of claim.

In a pecuniary interest is involved, the insurable interest requirement in life insurance can
met. Even when there is no relationship by blood or marriage, one person may be financially
harmed by the death of another. In other relationships, particularly those of a business nature,
the death of insured must give rise to the definite and measurable financial loss if insurable
interest is to exist. Examples include:

 The interest of a theatrical producer in life of an actor

 A professional football club in the lives outstanding players

 A corporation in lives of key employees

 A partner in the lives of other partner

 Creditors in lives of debtors

6.8. Fidelity and surety Bonds Insurance

 Dear student, what do you think is Fidelity and surety bonds?

___________________________________________________________________________

! There are two basic types of financial protection against the catastrophic losses that can

be caused by crime. These are:


1) Fidelity bonds
2) Surety bonds

1) Fidelity bonds provide protection against loss caused by the dishonest fraudulent acts of
employees such as embezzlement and theft of money. As such the bonds are hardly

118
distinguishable from insurance as far as the employer is considered. Although technically
there are three parts to fidelity bond-the employer (oblige), the employee (obligor), and the
insurer (surety)-in practice the main parties are only two the employer and the surety

2) Surety bonds provide for monetary compensation in case of failure by bonded persons to
perform certain acts, such as failure of a contractor to construct building on time. There are
three parties to surety bonds. These are;
a) Principal is the party who agrees to perform certain acts or fulfill certain obligations. For
example, sunshine construction may agree to build an office for the city of Adama. If the
company is required to obtain a performance bond before the construction is awarded
Sunshine construction would be known as principal.
b) Oblige is who is reimbursed for damages if the principal fails to perform. Thus, the city of
Adama would be reimbursed for damages if Sunshine Construction failed to complete the
building on time or according to contract specifications.
c) Surety is the part who agrees to answer for the debt, default, or obligation or another.
For example, Sunshine Construction may purchase a performance bond from Ethiopia
Insurance Corporation. If Sunshine Construction (principal) fails to perform the city of
Adama (Oblige) would be reimbursed for any loss by Ethiopian Insurance Corporation.

6.9. Engineering Insurance

 What is Engineering Insurance?

__________________________________________________________________________

! Engineering insurance provides protection against mechanical or electrical breakdown

of machinery and plant equipment such as lifts, boiler systems and electrical equipment in the
workplace. Cover extends to self-damage, damage to surrounding property, third party risks
and engineering interruption insurance. The principal risks covered are against explosion and
collapse, breakdown, sudden and unforeseen physical damage and fragmentation risk. This
insurance covers you against misfortunes such as machinery breakdown, business
interruption, deterioration of stock, engineering transit, dismantling and erection and

119
accidental damage to electronic equipment, mobile plant and construction work. All
engineering material damage policies are policies of indemnity. Naturally there are
definitions on these policies, which are a little out of the ordinary dictated by the type of
cover underwritten.

Engineering Insurance policies provide “All Risk” type covers. This means that almost any
sudden and unforeseen physical loss or damage occurring during the period of insurance to
the property insured is indemnifiable (payable by insurers.) In essence, every hazard is
covered which is not specifically excluded.

! Who Needs Engineering Insurance?

Financiers, Developers, Owners, Principals, Employers, Purchasers, Main Contractors, Sub-


Contractors, Supplier-Installers who are involved in:

 The construction or erection of all types of civil, electrical, mechanical engineering


projects.

 The managing and on-going operation of these types of industrial


developments/projects.

 The use/operating or leasing (hiring-out) of Contractor’s Plant and Machinery.

 Companies operating in the fields of civil, electronic, mechanical or structural


engineering.

Why should you take out Engineering Insurance?

 Unfortunately accidents can happen and there can be sudden and unforeseen loss or
damages to the Works/Projects under construction. Example: Brought about by a fire,
theft, storm, hail, collapse etc.

 There could be possibilities of resultant loss of gross profit, loss of rent, loss of
revenue, and/or increase cost of working etc.

 Consequential losses stemming from engineering damage in the form of loss of


profits, deterioration of stock, temporary hiring fees and increased cost of working

120
Engineering Insurance can include

a. Construction Risks:

Roads, bridges, factories, mines, airports, houses, buildings or power stations

b. Working Risks:

Computers, X-ray and scanning machines, mines, engineering works, glass works,
manufacturing operations, machinery of varying descriptions, clothing and furniture, food
and beverage processing, frozen foodstuffs and the meat industry.

 Activity 3

1) Distinguish between fidelity bonds and surety bonds?

___________________________________________________________________________
___________________________________________________________________________

2) You should need engineering insurance and why should they need?

___________________________________________________________________________
___________________________________________________________________________

 Check list

Dear learner, below are the most important points which are drawn from the preceding
chapter. Put a tick () mark in the box after the point you feel you have understood. But if
you find a concept that you have not mastered yet, please go back and read the chapter you
passed through.

I can:

1) Define motor/automobile insurance

2) Distinguish burglary, robbery, and theft insurance

3) Explain fire and lighting insurance

121
4) Identify risks included under ocean marine insurance

5) Explain aviation insurance and risks covered under it

6) Distinguish fidelity and surety bonds insurance

7) Explain engineering insurance

Chapter Summary
In the preceding chapter we have discussed many types of non-life insurance such as motor
insurance, fire and lighting insurance, marine insurance, aviation insurance, liability
insurance, engineering insurance, and so on.

Most motor or automobile insurance contracts are schedule contracts that permit the insured
to purchase both property and liability insurance under one policy. There are two types of
motor or automobile insurance-Private auto policy and commercial auto policy.
Fire and lightening insurance is designed to indemnify the insured for loss of, or damaged to,
buildings, furniture, fixtures, or other personal property as a result of fire, lighting, etc.
Ocean marine insurance provides protection for goods transported over water. Ocean marine
policies relate to three areas of risk: the hull, cargo and freight. And an aviation insurance
that provides protection against loss of damage to the different types of passenger and cargo
planes, and associated losses.
Liability insurance contracts obligate the insurer to pay amounts the insured becomes legally
obligated to pay as a result of covered injury to others. In addition, the insurer is obligated to
provide legal defense services for the insured against liability claims to which the coverage
applies.
There are two basic types of financial protection against the catastrophic losses that can be
caused by crime. These are: Fidelity bonds and Surety bonds
Engineering insurance provides protection against mechanical or electrical breakdown of
machinery and plant equipment such as lifts, boiler systems and electrical equipment in the
workplace.

122
 Self Test Questions

Part-I: Say “True” for correct statement and “False” for incorrect
statement
1) In automobile insurance both property and liability are insured under one policy.
2) Loss occurring in connection with bodily injury if the number of persons exceeds the
sitting capacity is compensated in private auto insurance.
3) According to Burglary policy of Ethiopian Insurance Corporation if it is cause by members
of the insured’s household it is not indemnified.
4) Fire intentionally caused by public authority is not indemnified.
5) In surety bonds, principle is the parties who agree for debt, default, or obligation of
another.
Part-II: Match Column “B” to Column “A”
“A” “B”
1) Ocean marine insurance A) Insurance for airplanes
2) Aviation insurance B) Insurance for items transported over water
3) Fidelity insurance C) Provides protection against loss caused by the dishonest
or fraudulent acts of employees
4) Surety insurance D) Provides protection in case of failure by bonded persons
5) Engineering insurance E) provides protection against mechanical or electrical
breakdown of machinery and plant equipment

123
CHAPTER SEVEN
RE-INSURANCE

Introduction
Dear learner, in the preceding chapter we have discussed about insurance; both life and health
insurance and non-life insurance in detail. In this chapter will see bout reinsurance that is
insurance that is purchased by an insurance company (the "ceding company" under the
arrangement) from one or more other insurance companies (the "reinsurer") as a means of
risk management, sometimes in practice including tax mitigation and other reasons described
for reinsurance. The ceding company and the reinsurer enter into a reinsurance agreement
which details the conditions upon which the reinsurer would pay a share of the claims
incurred by the ceding company. In this chapter will see meaning of reinsurance, reasons for
reinsurance, function of reinsurance, and types of reinsurance.

Objectives
After studying this chapter, student will be able to:
 Define reinsurance
 Justify why reinsurance is needed
 Explain the function of reinsurance
 Discuss type of reinsurance
 Identify the role players of reinsurance
7.1. Meaning of Re-Insurance

 Dear learner, would you guess what re-insurance means?

__________________________________________________________________________

! Reinsurance is the shifting part or the entire insurance originally written one insurer to

another insurer. The insurer that initially writes the business is called the Ceding Company.
The insurer that accepts part or all of the insurers from the ceding company is called
reinsurer. The amount of insurance retained by the ceding company for its own account is
called the retention or retention limit. The amount of the insurance ceded to the re-insurer

124
is known as cession. Finally, the re insurer in turn may obtain reinsurer for another insurer.
The process by which the reinsurer passes all risk to another reinsurer is known as
retrocession.

It is simple, really! Reinsurance is insurance for insurance companies. Let me explain.


A form of insurance, being the insurance of one insurer (the reinsured) by another insurer (the
reinsurer) by means of which the reinsured is indemnified for loss under insurance policies
issued by the reinsured to the public. A contract of reinsurance is a contract under which one
insurer agrees to indemnify another with respect to actual loss sustained under the latter’s
policy or policies of insurance.

Reinsurance had a very simple beginning. When a risk that was too large for the company to
handle safely was presented to an insurer, it began shop around for another insurance
company that was willing to take a portion of the risk in return for a portion of the premium.
A few current reinsurance operations are still conducted in this manner but they ever-present
danger that a devastating loss might occur before the reinsurance becomes effectively lead to
the development of modern reinsurance treaties.
7.2. Reason for Re-insurance

 Would you guess the reasons for re-insurance?

___________________________________________________________________________

! Usually an insurance company will buy reinsurance for one or more of the following
reasons. These are:
1. Market Capacity.
2. Stabilization of Result.
3. Catastrophe Protection.
4. Strengthening of its Financial Structure.
1) Market Capacity. There is a highly respected rule of thumb in the insurance industry.
Never expose more than 5% of a company’s capital and surplus on any one risk. But, what if,
that company is being pressured to offer larger policy limits to its customers in order to stay
in business. What then? Reinsurance of course! Reinsurance is a means by which an insurer
can write more business at higher limits than its own resources would otherwise permit.

125
2) Stabilization of Result. A second reason for reinsurance is for Stabilization of Results. To
put it another way, giving your investment people a level playing field in order to maximize
investment results. After all, investment income is a significant source of revenue for an
insurance company. In order to gain a maximum yield from its investments, a company’s
management team must have assurance that there will be no drastic calls for cash to pay
unexpected or abnormal losses. Reinsurance while it is not a banking transaction does fulfill a
financing function. In this case by providing for the amortization of insurance losses over
time. A few examples of how stabilization can be achieved include:

1. Control of the exposure-to-loss factors on each individual risk.

2. Control of the total of all accumulated losses arising during the year.

3. Adjusting the mix of business.

3. Catastrophe Protection. One of reinsurance’s main and customary roles has always been
that of providing Catastrophe Protection. If yours is a company writing homeowner’s
coverage in Florida, you would need protection from hurricanes. A property insurer in
California is concerned with earthquakes. A writer of professional liability is concerned with
large shock losses from unfavorable jury awards or from a change in the legal climate which
could leave it vulnerable to unexpected legislative or court imposed exposures or coverage
not originally intended.

4. Strengthening of its Financial Structure. Lastly reinsurance can be used to strengthen


the financial structure of a company by providing such things as:

1. Surplus relief

2. Improvement to the company’s financial statement

3. Improvement of Best’s and saving and protection ratings

4. Meeting the requirements of regulatory authorities.

5. Modification of the company’s tax obligations.

! Reinsurance is purchased for many reasons. Some of the key considerations include:

126
1. The size of the company

2. The financial strength of the company

3. The lines of business written

4. The Experience of the company

5. The ownership of the company (Stock or mutual)

6. The state or states of domicile or residence

7. The agenda of management or the stockholders

8. The need to grow or diversify with new product lines

9. The need for higher limits.

Functions of Reinsurance
Reinsurance provides three essential functions:
1) It offers the direct insurer greater security for its equity and solvency, as well as stable
results when unusual and major events occur, by covering the direct insurer above certain
ceilings or against accumulated individual commitments;
2) It allows insurers to increase their available capacity - i.e. the maximum amount they can
insure for a given loss or category of losses, by enabling them to underwrite policies covering
a larger number of risks, or larger risks, without excessively raising their administrative costs
and their need to cover their solvency margin and, therefore, their shareholders' equity;
3) It makes substantial liquid assets available to insurers in the event of exceptional losses.

In addition, reinsurers also provide advisory services to ceding companies by:

 defining their reinsurance needs and devising the most effective reinsurance program
to better plan their capital needs and solvency margin;

 supplying a wide array of support services, specifically in terms of technical training,


organization, accounting and information technology;

 providing expertise in certain highly specialized areas such as the analysis of complex
risks and risk pricing;

127
 enabling ceding companies to build up their business even if they are temporarily
under-capitalized, particularly in order to launch new products requiring heavy
investment.

7.3. Types of Re-insurance

 What are the types of re-insurance do you know?

___________________________________________________________________________

! There are two principals’ forms of reinsurance:

1) Facultative reinsurance and


2) Treaty insurance
1) Facultative reinsurance is an optional case-by-case method that is used when the ceding
company receives an application for insurance that exceeds its retention limit. Before the
policy is insure the primary insurer shops around for reinsurance and contacts several
reinsurers. A life insurer, for example may receive an application for Birr one million of life
insurers on a single life. Not wishing to reject this business but still unwilling to accept the
entire risk, the primary insurer communicates full details on this application to another
insurer with whom it has done business in the past. The other insurer may agree to assume 40
percent of any loss for a corresponding percentage of the premium. The primary insurer is
under an obligation to cede insurance, and the reinsurer is under no obligation to accept the
insurance. But if a willing reinsurance can be found, the primary insurer and reinsurer can
enter into a valid contract.

Facultative reinsurance is frequently used when a large amount of insurance is desired.


Before the application is accepted, the primary insurer determines if reinsurance can be
obtained. If available the policy can then be written.

Facultative reinsurance has the advantage of flexibility since reinsurance contract can be
arranged to fit any kind of case. It can increase the insurers’ capacity to write large amounts
of insurance. The reinsurance tends to stabilize the insurer’s operation by shifting large losses
to the reinsurer.

128
The major disadvantage of facultative reinsurance is that it is uncertainty. The ceding insurer
does not known in advance if reinsurer will accept any part of the insurance. There is also a
further disadvantage of delay, since the policy will not be issued until reinsurance is
obtained.

2) Treaty Reinsurance is also called automatic treaty means the primarily insurer has agreed
to cede insurance to the reinsurer, and the reinsurer has agreed to accept the business. All
business that falls within the scope of agreement is automatically reinsured according to the
terms of the treaty. Under automatic treaty reinsurers the ceding insurer agrees to pass on to
the reinsurer all business included within the scope the treaty, the reinsurer agrees to accept
this business, and the terms. Examples: the premium rates and the method of sharing the
insurance and the losses of the agreement.

Treaty reinsurance has several advantages to the primarily insurer. It is automatic and no
uncertainty or daily is involved. It is also economically, since it is not necessary to shop
around for reinsurance before the policy is written.
Treaty reinsurance could be unprofitable to the reinsurer. The reinsurer generally has no
knowledge about the insurance applicant and must really on the underwriting judgment of the
primarily insurer. The primarily insurer may written bad business and the insurer it. Also, the
premium received by the reinsurer may be inadequate. Thus, if the primarily insurer has a
poor selection of risks or charges in adequate rates, the reinsurer could occur loss.

! There are several types of reinsurance treaties and arrangements including the following:

1) Quota sharing treaty


2) Surplus share treaty
3) Excess of loss treaty
4) Reinsurance pool

1) Quota sharing treaty


Quota Share reinsurance is reinsurance in its most simplistic form. A quota share treaty
allows a reinsured such as XYZ to cede a fixed percentage of every risk that is written to a
group of reinsurers who are a party to such a quota share agreement.

129
Quota Share Example:

 80% Quota Share

 Reinsurer receives 80% of the premium less a commission which is retained by the
Reinsured to pay for its operating expenses.

 Reinsurer pays 80% of each and every loss no matter how big or small

2) Surplus share treaty


Under a surplus-share treaty, the reinsurer agrees to accept insurance in excess of the ceding
insurer’s limit, up to some maximum amount. The retention limit is referred to as a line and
is stated as a Birr amount. Under share treaty the ceding company decides what its net
retention will be for each class of business. The reinsurer does not participate unless the
policy exceeds the retention limit; the excess insurance is ceded to the reinsurer up to some
maximum limit. The primary insurer and reinsurer then share premiums and losses based on
the fraction of total insurance retained by each party.
3) Excess of loss treaty

Excess of loss reinsurance on the other hand grew primarily out of the requirement of the
original insurer (the Reinsured) for protection from a catastrophe or the need to issue high
limits of liability. Excess of loss reinsurance may be best described as an agreement whereby
the reinsurer agrees to reimburse the reinsured for all losses over a set birr amount.

The following example illustrates how the excess of loss treaty works. Assume that the
reinsurer agrees to pay for all losses in excess of birr 50,000 up to a further birr 200,000: the
way in which various losses are divided is shown below:

Loss Direct Insurer Excess Treaty

Birr 10,000 Birr 10,000 Nil

50,000 50,000 Nil

70,000 50,000 20,000

100,000 50,000 50,000

250,000 50,000 250,000

130
4) Reinsurance pool
Reinsurance can also be provided by a reinsurance pool. A reinsurance pool is an
organization of insurers that underwriters insurance on a joint basis. Reinsurance pools have
been formed because a single insurer alone may not have the financial capacity to write large
amounts of insurance, but the insurers as a group can combine their financial resources to
obtain the necessary capacity. For example, the combined hull and liability loss exposures
on a commercial jet can exceed birr 300 million if the jet should crash. Such high limits are
usually beyond the financial capacity of a single insurer. However, a reinsurance pool for
aviation can provide the necessary capacity.

7.4. The Insurance/Reinsurance Players

 Dear learner, can you guess the core players of Reinsurance?

___________________________________________________________________________

! The following are core players of reinsurance:

1) Agent – Insurance is often sold to the public through an Insurance Agent. This agent may
represent many insurance companies or only one or two. An agent representing many
companies is generally referred to as being part of the Independent Insurance Network. An
agent representing only one or two companies is generally referred to as a “Captive Agent”.
Often the latter is an employee of the insurance company he or she represents. An Agent will
be appointed by the insurance companies he/she represents and is considered the
representative of those insurance companies.

2) Broker – Technically a Broker represents the policy Holder who is their clients. A broker
will select an insurance company or several insurance companies and present his client needs
and exposures to them for quotations and ultimately binding of coverage. Like agents,
brokers are paid on a commission basis by the insurance company.

3) Direct Writer – Some insurance companies sell coverage on a “direct” basis. In other
words, employees of the insurance company deal directly with the prospective insured. The
application and other necessary underwriting details are submitted directly to the insurance

131
company which provides a quotation and will accept the order to bind coverage if acceptable.

4) Reinsurance Company – An insurance company that accepts the transfer of coverage and
risk from another insurance company. The original insurance company is commonly referred
to as the reinsured or ceding company. The company accepting the reinsurance is referred
to as the reinsurer. The transfer of such reinsurance is often called a cession.

5) Reinsurance Intermediary – A professional broker licensed to negotiate and transact


reinsurance contracts, cessions, and claims as necessary on behalf of a reinsured and a
reinsure or group of reinsurers.

 Activity 1

1) Explain re-insurance and its main reasons


___________________________________________________________________________
___________________________________________________________________________

2) Explain the advantages and disadvantages of facultative reinsurance


___________________________________________________________________________
___________________________________________________________________________
3) List and explain the major actors of reinsurance
___________________________________________________________________________
___________________________________________________________________________

 Check List
Dear learner, below are the most important points which are drawn from the preceding
chapter. Put a tick () mark in the box after the point you feel you have understood. But if
you find a concept that you have not mastered yet, please go back and read the chapter you
passed through.

I can:

1) Define Reinsurance

2) Distinguish ceding, cession, retrocession in reinsurance

3) Identify reasons for reinsurance

4) Explain types of reinsurance

5) Identify major actors of Reinsurance

132
Chapter Summary
Reinsurance is the shifting part or the entire insurance originally written by one insurer to
another insurer. The insurer that initially writes the business is called the Ceding Company.
The insurer that accepts part or all of the insurers from the ceding company is called
reinsurer. Usually an insurance company will buy reinsurance for one or more of the
following reasons. These are: market capacity, stabilization of result, catastrophe protection,
and strengthening of its financial structure.

There are two principals’ forms of reinsurance; Facultative reinsurance and Treaty insurance.
Facultative reinsurance is an optional case-by-case method that is used when the ceding
company receives an application for insurance that exceeds its retention limit. Treaty
Reinsurance is also called automatic treaty means the primarily insurer has agreed to cede
insurance to the reinsurer, and the reinsurer has agreed to accept the business. There are
several types of reinsurance treaties and arrangements including the following: quota sharing
treaty, surplus share treaty, excess of loss treaty, and reinsurance pool.
Agent, Broker, direct writer, Reinsurance Company, and Reinsurance intermediary are the
major players in reinsurance.

 Self Test Questions

Part-I: Say “True” for the correct statement and “False” for incorrect statement
1) The insurance company that initially writes the business of insurance is called the
reinsurer.
2) One of reinsurance’s main and customary roles has always been that of providing
Catastrophe Protection.
3) Reinsurance makes substantial liquid assets available to insurers in the event of
exceptional losses.
4) Flexibility is the main disadvantage of facultative reinsurance.
5) In surplus share treaty the retention limit is referred to as a line.
Part-II: Fill in the blank space/s the appropriate word or phrases
1) The process by which the reinsurer passes all risk to another reinsurer is known as_______
2) __________ is a type of reinsurance that an optional case-by-case method that is used
when the ceding company receives an application for insurance that exceeds its retention
limits. 3) What are the core players of reinsurance?

133
CHAPTER EIGHT
THE INSURANCE BUSINESS IN ETHIOPIA

Introduction

Dear student, in the preceding chapter we have discussed internationally applicable insurance
definitions, theories, principles, types, and reinsurance functions. What we have discussed in
aforementioned chapters about insurance also applicable for Insurance companies in
Ethiopia. Insurance industry in Ethiopia is in its infant stage. In this chapter will see briefly
development of insurance in Ethiopia, regulation of insurance companies in Ethiopia.

Objectives
After studying this chapter, student will be able to:
 Explain the development of insurance in Ethiopia
 Read the insurance provisions of Ethiopia

8.1. Development of Insurance in Ethiopia

 Dear learner, would you say something about Insurance industry in Ethiopia?

___________________________________________________________________________
Europeans have introduced modern forms of insurance services to Ethiopia. Indigenous self
help associations such as Ekub and Idir had existed earlier and have been used for centuries.
These traditional forms of social security due to their operational simplicity and their
voluntary nature are still active, widespread and strong in various parts of urban and rural
Ethiopia providing their members with financial and material assistance.

The practice of insurance services in its modern sense is a recent phenomenon, which is said
only to have been started in the early 1920’s. This was when the then Bank of Abyssinia
began to underwrite fire and marine insurance as an agency to a foreign company.

! The insurance business in Ethiopia in its modern application is a recent phenomenon. A

134
branch of a foreign company known as “Baloise Fire Insurance Company” was opened by an
Austrian (Weinsinger) in Addis Ababa in 1923. For the first time in Ethiopia the company
paid compensation to a client in 1929 for damage to his store caused by fire. Beginning from
this time until the Italian invasion of 1936 some foreign insurance companies were operating
through their agents. During the Italian occupation of Ethiopia in 1936-1941, Italian
insurance companies operated and non-Italian companies were closed down.

Despite all the achievements in the expansion of insurance services in the country, all of them
had their head offices abroad. An insurance company with its head office in Addis Ababa was
established for the first time in 1951. This company of Ethiopia gradually continued to
expand the insurance business from the capital city to the major towns of the country. Before
the Derg came to power, there were 13 insurance companies operating in the country. On
January 1975, Derg nationalized all banks and thirteen insurance companies that were
operating in the country. The boards of all the nationalized companies were dissolved and a
new provisional Insurance Board was set up. The nationalized companies were operating
independently but all were required to report to the Provisional Insurance Board.

8.2. Regulation of Insurance Companies

 Do you know any regulation related to Insurance companies?

___________________________________________________________________________

! The Ethiopia Insurance Corporation was established under proclamation No. 68/1975

with a paid up capital of 11 million dollars. The assets, liabilities, rights and obligations of
the nationalized private insurance companies were transferred to the Ethiopian Insurance
Corporation. Ethiopian Insurance Corporation was established with the following major
objectives. These are;
 To engage in all classes of insurance business in Ethiopia
 To ensure that the insurance services reach the broad masses of the people
 To promote efficient utilization of both material and finance insurance resources.

The Ethiopian Insurance Corporation operated as a sole insurance of organization until 1994.

135
! Dear student please read Proclamation No.68/1975 in order to know in depth insurance

industry of Ethiopia.

Insurance Business Since 1991


Following the change of government in 1991 a new economic policy that increased the role
of the private sector in the economy was formulated. A new and comprehensive law to
regulate the licensing operation and supervision of insurance business was promulgated by
the Transitional Government of Ethiopia under proclamation No. 86/1994. Under this
legislation the task of the licensing and supervision of insurance business was given to the
National Bank of Ethiopia.

! The law allowed private companies whose capital is wholly owned by Ethiopian

nationals and/or organizations wholly owned by Ethiopian nationals and registered under the
laws of and having their head office in Ethiopia to engage in insurance business.
Proclamation No. 86/1994 further provides that the minimum share capital is Birr 3 million
for general insurance business, Birr 4 million for long term insurance business, and Birr 7
million if the business to be done is both general and long term insurance business.

It is with this legal frame work that many private insurance companies enter in to Insurance
business as this time. Examples of Insurance companies currently operating in Ethiopia
include;
 Africa Insurance Sh.Co  Nib Insurance Sh.Co
 Awash Insurance Sh.Co  Nice Insurance Sh.Co
 Ethiopia Insurance Corporation  Nile Insurance Sh.Co
 Ethiopia-Life Insurance Company  Nyala Insurance Sh.Co
 Global Insurance Sh.Co  Oromia Insurance Sh.Co
 Lion Insurance Sh.Co  United Insurance Sh.Co
Except Ethiopia Insurance Corporation all the above mentioned insurance companies are
privately owned, and there are also many insurance companies under establishment.

136
 Activity 1

1) Explain history of Insurance development in Ethiopia.


___________________________________________________________________________
___________________________________________________________________________

2) Visit any insurance companies and ask any issues that you have not understood in this
module about insurance.
___________________________________________________________________________
___________________________________________________________________________

 Check List

Dear learner, below are the most important points which are drawn from the preceding
chapter. Put a tick () mark in the box after the point you feel you have understood. But if
you find a concept that you have not mastered yet, please go back and read the chapter you
passed through.

I can:

1) Understood the development of insurance in Ethiopia

2) Explain Insurance business in Ethiopia pre 1991 and post 1991

3) List many insurance companies in Ethiopia

4) Identify objectives of Ethiopian Insurance Corporation

Chapter Summary

Europeans have introduced modern forms of insurance services to Ethiopia. Indigenous self
help associations such as Ekub and Idir had existed earlier and have been used for centuries.
The insurance business in Ethiopia in its modern application is a recent phenomenon. A
branch of a foreign company known as “Baloise Fire Insurance Company” was opened by an
Austrian (Weinsinger) in Addis Ababa in 1923.

The Ethiopia Insurance Corporation was established under proclamation No. 68/1975 with a

137
paid up capital of 11 million dollars. The Ethiopian Insurance Corporation operated as a sole
insurance of organization until 1994. Following the change of government in 1991 a new
economic policy that increased the role of the private sector in the economy was formulated.
A new and comprehensive law to regulate the licensing operation and supervision of
insurance business was promulgated by the Transitional Government of Ethiopia under
proclamation No. 86/1994. Currently numbers of insurance companies are increasing
especially from private investors and there are one public and more than ten private insurance
companies in Ethiopia.

 Self Test Questions

Part-I: Say “True” for the correct statement and “False” for incorrect
statement.
1) Indigenous self help associations such as Ekub and Idir had existed earlier and have been
used for centuries but now there are no such associations.

2) Insurance business Ethiopia is in its boom stage.

3) The Ethiopia Insurance Corporation was established under proclamation No. 68/1994.

4) Compensation by Insurance Company in Ethiopia was first paid in 1929.

5) Ethiopian societies have great awareness of Insurance.

Part-II: Short Answer Questions


1) When insurance was started in Ethiopia?

2) The Only public Insurance Company in Ethiopia is________


References
 Teklegiorgis Assefa (2004). Principle of Risk management & Insurance, Mekelle,
Ethiopia

 Rejda E. George (1997). Principles of risk management & insurance, 6th ed.,
Library of congress cataloging in production, USA.
 Williams, Jr C.A Michael L.S & Peter C. young (1998), Risk management &
Insurance ,8 t h ed., Mc graw Hill
 Trieschmann kames S. Robert E. Hoyt & David W. Sommer (2005). Risk
management & Insurance, 12th ed. South –western, India.

138
 Key Answer to Self Test Questions
Chapter-one
Part-II
Part-I 1) B
1) False 2) E
2) True 3) C
3) False 4) D
4) False 5) B
5) False
Part-II
1) The most widely held meaning of uncertainty refers to a state of mind characterized by
doubt, based on lack of knowledge about what will or will not happen in the future.
Uncertainty, then, is simply a psychological relation to the absence of knowledge about the
future. In contrast risk is a situation where there is a possibility of loss and no loss and even a
possibility of profit (although it is not always and in every risky situation). Risk can be
objective but uncertainty is always subject and there is no parameter that measure
uncertainty.
2) Peril is a cause of loss. It is a contingency that cause a loss. Examples, fire, flood, theft,
collusion, earth quake, etc
Hazard is a condition or a situation that increases chance of loss resulting from a given peril.
Examples, unlocked door increases chance of theft, defective wiring increases chance of fire,
etc
3) Pure risk defined as a situation in which there are only the possibilities of loss or no loss.
Speculative risk is defined as a situation in which either profit or loss is possible.
4) Direct loss is defined as a financial loss that results from the physical damage, destruction,
or theft of property. Indirect /consequential loss is a financial loss results indirectly from the
occurrence of direct loss.

Chapter Two 5) False 1) C


Part-I 6) False 2) A
1) False 7) False 3) E
2) False 8) False 4) D
3) True 9) True 5) C
4) True 10) True
Part-II
Part-III
1) Pre-loss objectives include; economy, reduction in anxiety, and meeting external
obligations and post-loss objectives include; survival, continuity of operations, earning
stability, continued growth, and social responsibility.
2) The four categories of risk exposure include: Physical, financial, liability, and human
exposures.

139
3) The four steps in risk management process:
1) Identification of potential losses
2) Evaluation of potential losses
3) Selecting the appropriate techniques of risk management
4) Implementation and administration of the program
4) What are several important factors are to be considered by the risk manager in selection of
insurer. These include the following:
 Financial strength of the insurer
 Risk management services provided by the insurer
 The cost and terms of product
5) Self insurance plan are distinguished from other insurance operations by having the
pooling of exposures and funding of the cost of losses take place within one business entity

Chapter Three
Part-I 6) True
1) True 7) True
2) False 8) False
3) False 9) True
4) False 10) True
5) False
Part-II
1) The basic characteristics of insurance include the following:
1) Pooling of losses
2) Payment of fortuitous losses
3) Risk transfer
4) Indemnification
(Refer your module for their explanation)
2) The six requirements of an insurable risk are:
7. There must be a large number of homogeneous exposure units
8. The loss must be accidental and unintentional
9. The loss must be determinable and measurable.
10. The loss should not the catastrophic
11. The chance of loss must be calculable
12. The premium must be economically feasible
3) The costs of insurance to the society include; operating expenses, fraudulent claim, and
inflated claim. Inflated claim is an important social cost of insurance because premium must
be increased to cover the loss and disposable income and consumption of other goods or
services is thereby reduced.

140
Chapter Four
Part-I Part-II
1) False 1) A
2) True 2) B
3) False 3) B
4) True 4) C
5) False 5) B
Chapter Five
Part-I Part-II
1) False 6) True 1) E
2) False 7) True 2) A
3) True 8) False 3) A
4) False 9) True 4) B
5) False 10) True 5) C
Part-III
1) Pertinent information needed for underwriting is obtained from the following sources:
 Proposal Form  Questionnaires and Interview
 Medical Report  Underwriting Manuals
 Attending Physicians Statement  Inspection Report
 Agent’s/ Salesman’s Report
2) Medical expenses insurance is paid under the following specific coverage’s
5. Hospital insurance 7. Physicals expense insurance
6. Surgical insurance 8. Major medical insurance.
3) There are three major types of life insurance. These are term, endowment, and whole life
insurance. The selection is up to you and your justification matters.
Chapter Six
Part-I Part-II
1) True 1) B
2) False 2) A
3) True 3) C
4) True 4) D
5) True 5) E
Chapter Seven
Part-I 3) True
1) False 4) False
2) True 5) True

141
Part-II:
1) Retrocession
2) Facultative reinsurance
3) Agent, Broker, direct writer, Reinsurance Company, and Reinsurance intermediary

Chapter Eight
Part-I 3) False
1) False 4) True
2) False 5) False
Part-II
1) Insurance Company in Ethiopia was started when a branch of a foreign company known
as “Baloise Fire Insurance Company” was opened by an Austrian (Weinsinger) in Addis
Ababa in 1923.
2) Ethiopian Insurance Corporation

142
143

Das könnte Ihnen auch gefallen